You are on page 1of 118

Many electric circuits are complex, but it is an engineers goal to reduce their complexity to analyze them easily.

In the previous chapters, we have mastered the ability to solve networks containing independent and dependent sources making use of either mesh or nodal analysis. In this chapter, we will introduce new techniques to strengthen our armoury to solve complicated networks. Also, these new techniques in many cases do provide insight into the circuits operation that cannot be obtained from mesh or nodal analysis. Most often, we are interested only in the detailed performance of an isolated portion of a complex circuit. If we can model the remainder of the circuit with a simple equivalent network, then our task of analysis gets greatly reduced and simplied. For example, the function of many circuits is to deliver maximum power to load such as an audio speaker in a stereo system. Here, we develop the required relationship betweeen a load resistor and a xed series resistor which can represent the remaining portion of the circuit. Two of the theorems that we present in this chapter will permit us to do just that.

3.1

Superposition theorem

The principle of superposition is applicable only for linear systems. The concept of superposition can be explained mathematically by the following response and excitation principle :
i1 i2

then;

i1

+ i2

! ! !

v1 v2 v1

+ v2

The quantity to the left of the arrow indicates the excitation and to the right, the system response. Thus, we can state that a device, if excited by a current i1 will produce a response v1 . Similarly, an excitation i2 will cause a response v2 . Then if we use an excitation i1 + i2 , we will nd a response v1 + v2 . The principle of superposition has the ability to reduce a complicated problem to several easier problems each containing only a single independent source.

160

Network Theory

Superposition theorem states that, In any linear circuit containing multiple independent sources, the current or voltage at any point in the network may be calculated as algebraic sum of the individual contributions of each source acting alone. When determining the contribution due to a particular independent source, we disable all the remaining independent sources. That is, all the remaining voltage sources are made zero by replacing them with short circuits, and all remaining current sources are made zero by replacing them with open circuits. Also, it is important to note that if a dependent source is present, it must remain active (unaltered) during the process of superposition. Action Plan: (i) In a circuit comprising of many independent sources, only one source is allowed to be active in the circuit, the rest are deactivated (turned off). (ii) To deactivate a voltage source, replace it with a short circuit, and to deactivate a current source, replace it with an open circuit. (iii) The response obtained by applying each source, one at a time, are then added algebraically to obtain a solution. Limitations: Superposition is a fundamental property of linear equations and, therefore, can be applied to any effect that is linearly related to the cause. That is, we want to point out that, superposition principle applies only to the current and voltage in a linear circuit but it cannot be used to determine power because power is a non-linear function.
EXAMPLE 3.1

Find the current in the 6 resistor using the principle of superposition for the circuit of Fig. 3.1.

Figure 3.1
SOLUTION

As a rst step, set the current source to zero. That is, the current source appears as an open circuit as shown in Fig. 3.2. 6 6 = A i1 = 3+6 9

Circuit Theorems

161

As a next step, set the voltage to zero by replacing it with a short circuit as shown in Fig. 3.3. 2 3 6 i2 = = A 3+6 9

Figure 3.2

Figure 3.3

The total current i is then the sum of i1 and i2


i

= i1 + i2 =

12 A 9

EXAMPLE

3.2

Find io in the network shown in Fig. 3.4 using superposition.

Figure 3.4
SOLUTION

As a rst step, set the current source to zero. That is, the current source appears as an open circuit as shown in Fig. 3.5.

Figure 3.5

162

Network Theory

6 = 0:3 mA (8 + 12) 103 As a second step, set the voltage source to zero. This means the voltage source in Fig. 3.4 is replaced by a short circuit as shown in Figs. 3.6 and 3.6(a). Using current division principle,
io
0

iA

iR2 R1

+ R2

where

R1

= (12 k 12 k) + 12 k = 6 k + 12 k = 18 k

jj

= 12 k 4 10 3 12 103 iA = (12 + 18) 103 = 1:6 mA Again applying the current division principle, 12 iA 00 io = = 0:8 mA 12 + 12

and

R2

  

Figure 3.6

Thus;

io

= io 0 + io 00 =

0:3 + 0:8 = 0:5 mA

Figure 3.6(a)

Circuit Theorems

163

EXAMPLE

3.3

Use superposition to nd io in the circuit shown in Fig. 3.7.

Figure 3.7

SOLUTION

As a rst step, keep only the 12 V source active and rest of the sources are deactivated. That is, 2 mA current source is opened and 6 V voltage source is shorted as shown in Fig. 3.8. 12 (2 + 2) 103 = 3 mA =

io

Figure 3.8

As a second step, keep only 6 V source active. Deactivate rest of the sources, resulting in a circuit diagram as shown in Fig. 3.9.

164

Network Theory

Applying KVL clockwise to the upper loop, we get

 10

io

00

 10
00

io

00

io

6=0 6 = 1:5 mA 103

Figure 3.9

As a nal step, deactivate all the independent voltage sources and keep only 2 mA current source active as shown in Fig. 3.10.

Figure 3.10

Current of 2 mA splits equally. Hence;


io
000

= 1mA

Applying the superposition principle, we nd that


io

= io 0 + io 00 + io 000 =3 1:5 + 1 = 2:5 mA

Circuit Theorems

165

EXAMPLE

3.4

Find the current i for the circuit of Fig. 3.11.

Figure 3.11
SOLUTION

We need to nd the current i due to the two independent sources. As a rst step in the analysis, we will nd the current resulting from the independent voltage source. The current source is deactivated and we have the circuit as shown as Fig. 3.12. Applying KVL clockwise around loop shown in Fig. 3.12, we nd that

5i1 + 3i1

24 = 0 24 i1 = = 3A 8
i2

As a second step, we set the voltage source to zero and determine the current current source. For this condition, refer to Fig. 3.13 for analysis.

due to the

Figure 3.12

Figure 3.13

Applying KCL at node 1, we get


i2

+7=
i2 v1

v1

3i2 2 0

(3.1)

Noting that we get,

= =

v1

3 3i2

(3.2)

166

Network Theory

Making use of equation (3.2) in equation (3.1) leads to 3i2 3i2 i2 + 7 = 2 7 i2 = A 4 Thus, the total current

= i1 + i2 7 5 =3 A= A 4 4

EXAMPLE

3.5

For the circuit shown in Fig. 3.14, nd the terminal voltage Vab using superposition principle.

SOLUTION

Figure 3.14

As a rst step in the analysis, deactivate the independent current source. This results in a circuit diagram as shown in Fig. 3.15. Applying KVL clockwise gives 4 + 10

0+3 ) )

Vab1

+ Vab1 = 0 4Vab1 = 4
Vab1

= 1V
Figure 3.15

Next step in the analysis is to deactivate the independent voltage source, resulting in a circuit diagram as shown in Fig. 3.16. Applying KVL gives 10

2+3 ) )

Vab2

+ Vab2 = 0 4Vab2 = 20
Vab2

= 5V

Figure 3.16

Circuit Theorems

167

According to superposition principle,


Vab

= Vab1 + Vab2 = 1 + 5 = 6V

EXAMPLE

3.6

Use the principle of superposition to solve for vx in the circuit of Fig. 3.17.

Figure 3.17

SOLUTION

According to the principle of superposition,


vx

= vx 1 + vx 2

where vx1 is produced by 6A source alone in the circuit and vx2 is produced solely by 4A current source. To nd vx1 , deactivate the 4A current source. This results in a circuit diagram as shown in Fig. 3.18. KCL at node x1 :
vx 1 vx 1

2 But Hence;
vx 1

4ix1 8
ix1

=6 =
vx 1

2
vx 1

+ +

vx 1

v 1 4 x 2

8
vx 1

=6 =6
Figure 3.18

) ) )

2vx1 8

4vx1 + vx1
vx 1

2vx1 = 48 = 48 = 16V 3

168

Network Theory

To nd vx2 , deactivate the 6A current source, resulting in a circuit diagram as shown in Fig. 3.19. KCL at node x2 :
vx 2

) )

2
vx 2 vx 2 vx 2

( 4ix2 ) =4 8 2 vx 2 vx + 4ix2 + 2 =4 8 2 +
v x2

(3.3)

Applying KVL along dotted path, we get


vx 2 vx 2

+ 4ix2 2ix2 or

2ix2 = 0
ix2

vx 2

(3.4)

Substituting equation (3.4) in equation (3.3), we get +4


vx 2 vx 2 vx 2

) ) ) )

+ 8 +

2 2vx2 2
vx 2

=4 =4 =4

8
vx 2

4vx2 = 32 32 vx 2 = V 3

Hence, according to the superposition principle,


vx

= vx 1 + vx 2 32 = 16 = 5:33V 2

Figure 3.19

EXAMPLE

3.7

Which of the source in Fig. 3.20 contributes most of the power dissipated in the 2 resistor ? The least ? What is the power dissipated in 2 resistor ?

Figure 3.20

Circuit Theorems

169

SOLUTION

The Superposition theorem cannot be used to identify the individual contribution of each source to the power dissipated in the resistor. However, the superposition theorem can be used to nd the total power dissipated in the 2 resistor.

Figure 3.21

According to the superposition principle,


i1
0 = i0 1 + i2

where i0 1 = Contribution to i1 from 5V source alone. 0 and i2 = Contribution to i1 from 2A source alone. Let us rst nd i0 1 . This needs the deactivation of 2A source. Refer to Fig. 3.22. 5 = 1:22A 2 + 2:1 Similarly to nd i0 2 we have to disable the 5V source by shorting it.
i1
0

Referring to Fig. 3.23, we nd that


i2
0

2 2:1 = 2 + 2:1

1:024 A

Figure 3.22

Figure 3.23

170

Network Theory

Total current,
i1
0 = i0 1 + i2

= 1:22 Thus;
P2

1:024

= 0:196 A = (0:196)2

2

= 0:0768 Watts = 76:8 mW


EXAMPLE 3.8

Find the voltage V1 using the superposition principle. Refer the circuit shown in Fig.3.24.

Figure 3.24

SOLUTION

According to the superposition principle,


V1

= V10 + V100

where V10 is the contribution from 60V source alone and V100 is the contribution from 4A current source alone. To nd V10 , the 4A current source is opened, resulting in a circuit as shown in Fig. 3.25.

Figure 3.25

Circuit Theorems

171

Applying KVL to the left mesh: 30ia Also 60 + 30 (ia


ib ) ib

=0 = = 0:4iA 0:4 (
ia )

(3.5) = 0:4ia (3.6)

Substituting equation (3.6) in equation (3.5), we get

)
Hence;

30ia

60 + 30ia

30 0:4ia = 0 60 ia = = 1:25A 48 1:25 ib = 0:4ia = 0:4

= 0:5A
V1
0

= (ia

ib )

 30

= 22:5 V To nd, V100 , the 60V source is shorted as shown in Fig. 3.26.

Figure 3.26

Applying KCL at node a:


Va

)
Applying KCL at node b:

20 30Va

Va

=4 10 20V100 = 800

V1

00

(3.7)

V1

00

30 Also; Hence;
Va

V1

00

Va

= 20ia
V1
00

20 0:4Va + = 30 10 20 7:2Va + 8V100 = 0


V1
00

10
ib

= 0:4ib =
Va

Va

(3.8)

172

Network Theory

Solving the equations (3.7) and (3.8), we nd that


V1
00

= 60V = V10 + V100 = 22:5 + 60 = 82:5V

Hence

V1

EXAMPLE

3.9

(a) Refer to the circuit shown in Fig. 3.27. Before the 10 mA current source is attached to terminals x y , the current ia is found to be 1.5 mA. Use the superposition theorem to nd the value of ia after the current source is connected. (b) Verify your solution by nding ia , when all the three sources are acting simultaneously.

Figure 3.27
SOLUTION

According to the principle of superposition,


ia

= ia1 + ia2 + ia3

where ia1 , ia2 and ia3 are the contributions to ia from 20V source, 5 mA source and 10 mA source respectively. As per the statement of the problem,
ia1

+ ia2 = 1:5 mA

To nd ia3 , deactivate 20V source and the 5 mA source. The resulting circuit diagram is shown in Fig 3.28. 10mA 2k = 1 mA ia3 = 18k + 2k Hence, total current

ia

= ia1 + ia2 + ia3 = 1:5 + 1 = 2:5 mA

Circuit Theorems

173

Figure 3.28

(b) Refer to Fig. 3.29 KCL at node y: 18

Vy

103

Vy

20 = (10+5) 10 103
Vy

Solving, we get Hence;


ia

18

Vy

= 45V: 45 = 3 10 18 103 = 2:5 mA

Figure 3.29

3.2

Thevenins theorem

In section 3.1, we saw that the analysis of a circuit may be greatly reduced by the use of superposition principle. The main objective of Thevenins theorem is to reduce some portion of a circuit to an equivalent source and a single element. This reduced equivalent circuit connected to the remaining part of the circuit will allow us to nd the desired current or voltage. Thevenins theorem is based on circuit equivalence. A circuit equivalent to another circuit exhibits identical characteristics at identical terminals.

Figure 3.30 A Linear two terminal network

Figure 3.31 The Thevenins equivalent circuit

According to Thevenins theorem, the linear circuit of Fig. 3.30 can be replaced by the one shown in Fig. 3.31 (The load resistor may be a single resistor or another circuit). The circuit to the left of the terminals x y in Fig. 3.31 is known as the Thevenins equivalent circuit.

174

Network Theory

The Thevenins theorem may be stated as follows: A linear twoterminal circuit can be replaced by an equivalent circuit consisting of a voltage source Vt in series with a resistor Rt , Where Vt is the opencircuit voltage at the terminals and Rt is the input or equivalent resistance at the terminals when the independent sources are turned off or Rt is the ratio of opencircuit voltage to the shortcircuit current at the terminal pair. Action plan for using Thevenins theorem : 1. Divide the original circuit into circuit A and circuit B .

In general, circuit B is the load which may be linear or non-linear. Circuit A is the balance of the original network exclusive of load and must be linear. In general, circuit A may contain independent sources, dependent sources and resistors or other linear elements.

2. Separate the circuit A from circuit B . 3. Replace circuit A with its Thevenins equivalent. 4. Reconnect circuit B and determine the variable of interest (e.g. current i or voltage v ).

Procedure for nding Rt : Three different types of circuits may be encountered in determining the resistance, Rt : (i) If the circuit contains only independent sources and resistors, deactivate the sources and nd Rt by circuit reduction technique. Independent current sources, are deactivated by opening them while independent voltage sources are deactivated by shorting them.

Circuit Theorems

175

(ii) If the circuit contains resistors, dependent and independent sources, follow the instructions described below: (a) Determine the open circuit voltage voc with the sources activated. (b) Find the short circuit current isc when a short circuit is applied to the terminals a (c)
Rt b

voc isc

(iii) If the circuit contains resistors and only dependent sources, then (a)
voc

= 0 (since there is no energy source)

(b) Connect 1A current source to terminals a b and determine vab . (c)


Rt

vab

Figure 3.32

For all the cases discussed above, the Thevenins equivalent circuit is as shown in Fig. 3.32.
EXAMPLE 3.10

Using the Thevenins theorem, nd the current i through R = 2 . Refer Fig. 3.33.

Figure 3.33
SOLUTION

Figure 3.34

176

Network Theory

Since we are interested in the current i through R, the resistor R is identied as circuit B and the remainder as circuit A. After removing the circuit B, circuit A is as shown in Fig. 3.35.

Figure 3.35

To nd Rt , we have to deactivate the independent voltage source. Accordingly, we get the circuit in Fig. 3.36.
Rt

= (5 20 ) + 4 = 5 20 +4=8 5 + 20

jj

Rt

Referring to Fig. 3.35, 50 + 25I = 0 Hence


Vab

= 2A

Figure 3.36

= Voc = 20(I ) = 40V

Thus, we get the Thevenins equivalent circuit which is as shown in Fig.3.37.

Figure 3.37

Figure 3.38

Reconnecting the circuit B to the Thevenins equivalent circuit as shown in Fig. 3.38, we get
i

40 = 4A 2+8

Circuit Theorems

177

EXAMPLE

3.11

(a) Find the Thevenins equivalent circuit with respect to terminals a b for the circuit shown in Fig. 3.39 by nding the open-circuit voltage and the shortcircuit current. (b) Solve the Thevenin resistance by removing the independent sources. Compare your result with the Thevenin resistance found in part (a).

Figure 3.39
SOLUTION

Figure 3.40

(a) To nd Voc : Apply KCL at node 2 :


V2

)
Hence;

60 + 20

30 40 V2 = 60 Volts +
V2

1:5 = 0

Voc

=I =

60 60 + 20 60 = 60 = 45 V 80

60   0 V
2

178

Network Theory

To nd isc :

Applying KCL at node 2:

V2

)
Therefore;

20

40 V2 = 30V = =
V2

V2

30

1:5 = 0

isc Rt

20
Voc isc

= 1:5A = 45 1:5
Figure 3.40 (a)

= 30 The Thevenin equivalent circuit with respect to the terminals a b is as shown in Fig. 3.40(a). (b) Let us now nd Thevenin resistance Rt by deactivating all the independent sources,

Rt

Rt

Rt

= 60 (40 + 20) 60 = = 30 (veried) 2

jj

It is seen that, if only independent sources are present, it is easy to nd Rt by deactivating all the independent sources.

Circuit Theorems

179

EXAMPLE

3.12
b.

Find the Thevenin equivalent for the circuit shown in Fig. 3.41 with respect to terminals a

Figure 3.41
SOLUTION

To nd Voc = Vab : Applying KVL around the mesh of Fig. 3.42, we get

20 + 6i

2i + 6i = 0
i

= 2A

Since there is no current owing in 10 resistor, Voc = 6i = 12 V To nd Rt : (Refer Fig. 3.43) Since both dependent and independent sources are present, Thevenin resistance is found using the relation,
Rt

Figure 3.42

voc isc

Applying KVL clockwise for mesh 1 :

)
Since i = i1
i2 ,

20 + 6i1

2i + 6 (i1 12i1

i2 )

=0

6i2 = 20 + 2i

we get 6i2 = 20 + 2 (i1 4i2 = 20


i2 )

) )

12i1 10i1

Applying KVL clockwise for mesh 2 : 10i2 + 6 (i2


i1 )

=0
Figure 3.43

6i1 + 16i2 = 0

180

Network Theory

Solving the above two mesh equations, we get


i2 Rt

= =

120 A 136
voc isc

isc

= i2 =

120 A 136

12 = 13:6 120 136

EXAMPLE

3.13

Find Vo in the circuit of Fig. 3.44 using Thevenins theorem.

Figure 3.44
SOLUTION

To nd Voc : Since we are interested in the voltage across 2 k resistor, it is removed from the circuit of Fig. 3.44 and so the circuit becomes as shown in Fig. 3.45.

Figure 3.45

By inspection, Applying KVL to mesh 2 :

i1

= 4 mA

12 + 6

 10

12 + 6
3

 10
i2

(i2

 10

i1 )

 + 3  103 i2 = 0 3
+3

 10

i2

=0

Circuit Theorems

181

Solving, we get

Applying KVL to the path 4 k

 10

i1

Voc

! a b ! 3 k, we get + 3  10 = 0 = 4  10 + 3  10 = 4  10  4  10 + 3  10  4  10
Voc

i2

= 4 mA

i2

3 3

i1

i2

= 28V

To nd Rt : Deactivating all the independent sources, we get the circuit diagram shown in Fig. 3.46.

Figure 3.46
Rt

= Rab = 4 k + (6 k 3 k) = 6 k

jj

Hence, the Thevenin equivalent circuit is as shown in Fig. 3.47.

Figure 3.47

Figure 3.48

If we connect the 2 k resistor to this equivalent network, we obtain the circuit of Fig. 3.48.
Vo

=i 2

 10  28  2  10 = (6 + 2)  10
3 3

= 7V

EXAMPLE

3.14

The wheatstone bridge in the circuit shown in Fig. 3.49 (a) is balanced when R2 = 1200 . If the galvanometer has a resistance of 30 , how much current will be detected by it when the bridge is unbalanced by setting R2 to 1204 ?

182

Network Theory

Figure 3.49(a)
SOLUTION

To nd Voc : We are interested in the galavanometer current. Hence, it is removed from the circuit of Fig. 3.49 (a) to nd Voc and we get the circuit shown in Fig. 3.49 (b). 120 120 = A i1 = 900 + 600 1500 120 120 i2 = = A 1204 + 800 2004 Applying KVL clockwise along the path 1204 b a 900 , we get

1204i2
Vt

Vt

900i1 = 0

900i1 120 900 = 1204 2004 = 95:8 mV

= 1204i2

120  1500
Figure 3.49(b)
a b

To nd Rt : Deactivate all the independent sources and look into the terminals Thevenins resistance.

to determine the

Figure 3.49(c)

Figure 3.49(d)

Circuit Theorems

183

Rt

= Rab = 600 900 + 800 1204 900 600 1204 800 = + 1500 2004 = 840:64

jj

jj

Hence, the Thevenin equivalent circuit consists of the 95.8 mV source in series with 840.64 resistor. If we connect 30 resistor (galvanometer resistance) to this equivalent network, we obtain the circuit in Fig. 3.50. 95:8 10 3 840:64 + 30

Figure 3.50

iG

= 110:03 A

EXAMPLE

3.15
a and b.

For the circuit shown in Fig. 3.51, nd the Thevenins equivalent circuit between terminals

Figure 3.51
SOLUTION

With ab shorted, let Isc = I . The circuit after transforming voltage sources into their equivalent current sources is as shown in Fig 3.52. Writing node equations for this circuit, At a : At c : At b : 0:2Va 0:1 Vc + I = 3 0:1 Vb = 4
I

0:1Va + 0:3 Vc

0:1Vc + 0:2 Vb

=1
Vb

As the terminals a and b are shorted Va = and the above equations become

Figure 3.52

184

Network Theory

0:2Va 0:2Va

0:1 Vc + I = 3 0:1 Vc 1=1

0:2Va + 0:3 Vc = 4

Solving the above equations, we get the short circuit current, I = Isc = 1 A. Next let us open circuit the terminals a and b and this makes I = 0. And the node equations written earlier are modied to 0:2Va 0:1Va + 0:3 Vc 0:1 Vc = 3 0:1 Vb = 4

0:1Vc + 0:2 Vb = 1 Solving the above equations, we get


Va

= 30V and Vb = 20V

20 = 10 V = Voc = Vt 10 Therefore Rt = = = 10 Isc 1 The Thevenins equivalent is as shown in Fig 3.53


Voc

Hence, Vab = 30

Figure 3.53

EXAMPLE

3.16
b.

Refer to the circuit shown in Fig. 3.54. Find the Thevenin equivalent circuit at the terminals a

Figure 3.54
SOLUTION

To begin with let us transform 3 A current source and 10 V voltage source. This results in a network as shown in Fig. 3.55 (a) and further reduced to Fig. 3.55 (b).

Circuit Theorems

185

Figure 3.55(a)

Again transform the 30 V source and following the reduction procedure step by step from Fig. 3.55 (b) to 3.55 (d), we get the Thevenins equivalent circuit as shown in Fig. 3.56.

Figure 3.55(b)

Figure 3.55(c)

Figure 3.55(d)

Figure 3.56 Thevenin equivalent


circuit

EXAMPLE

3.17
b.

Find the Thevenin equivalent circuit as seen from the terminals a shown in Fig. 3.57.

Refer the circuit diagram

186

Network Theory

SOLUTION

Figure 3.57

Since the circuit has no independent sources, i = 0 when the terminals a b are open. Therefore, Voc = 0. The onus is now to nd Rt . Since Voc = 0 and isc = 0, Rt cannot be determined from
Rt

Voc isc

. Hence, we choose to connect a source of 1 A at the terminals a


Rt

as shown in Fig.

3.58. Then, after nding Vab , the Thevenin resistance is, =


Vab

1 1=0

KCL at node a :

Va

2i 5
i

+ = +

Va

10
Va

Also; Hence;
Va

)
Hence;

2 5

Va 10

10
Va

10 50 Va = V 13
Rt

1=0

50 1 13 Alternatively one could nd Rt by connecting a 1V source at the terminals a b and then nd 1 . The concept of nding Rt by connecting a 1A source the current from b to a. Then Rt = =
Va

between the terminals a b may also be used for circuits containing independent sources. Then set all the independent sources to zero and use 1A source at the terminals a b to nd Vab and . hence, Rt = 1 For the present problem, the Thevenin equivalent circuit as seen between the terminals a is shown in Fig. 3.58 (a).
Vab b

iba

Figure 3.58

Figure 3.58 (a)

Circuit Theorems

187

EXAMPLE

3.18
b

Determine the Thevenin equivalent circuit between the terminals a

for the circuit of Fig. 3.59.

Figure 3.59

SOLUTION

As there are no independent sources in the circuit, we get Voc = Vt = 0: To nd Rt , connect a 1V source to the terminals a b and measure the current I that ows from b to a. (Refer Fig. 3.60 a). 1 Rt =
I

Figure 3.60(a)

Applying KCL at node a:


I

= 0:5Vx + = 1V = 0:5 +

Vx

Since; we get, Hence;

Vx I

1 = 0:75 A 4

1 Rt = = 1:33 0:75

Figure 3.60(b)

The Thevenin equivalent circuit is shown in 3.60(b). Alternatively, sticking to our strategy, let us connect 1A current source between the terminals
a b

and then measure Vab (Fig. 3.60 (c)). Consequently, Rt =

Vab

= Vab :

188

Network Theory

Applying KCL at node a:


Vx

0:5Vx + Hence
Rt

4
Vab

=1 =

Vx

= 1:33V

Vx

= 1:33

The corresponding Thevenin equivalent circuit is same as shown in Fig. 3.60(b)

Figure 3.60(c)

3.3

Nortons theorem

An American engineer, E.L. Norton at Bell Telephone Laboratories, proposed a theorem similar to Thevenins theorem. Nortons theorem states that a linear two-terminal network can be replaced by an equivalent circuit consisting of a current source iN in parallel with resistor RN , where iN is the short-circuit current through the terminals and RN is the input or equivalent resistance at the terminals when the independent sources are turned off. If one does not wish to turn off the independent sources, then RN is the ratio of open circuit voltage to shortcircuit current at the terminal pair.

Figure 3.61(a) Original circuit

Figure 3.61(b) Nortons equivalent circuit

Figure 3.61(b) shows Nortons equivalent circuit as seen from the terminals a b of the original circuit shown in Fig. 3.61(a). Since this is the dual of the Thevenin circuit, it is clear that voc . In fact, source transformation of Thevenin equivalent circuit leads to RN = Rt and iN = Nortons equivalent circuit. Procedure for nding Nortons equivalent circuit: (1) If the network contains resistors and independent sources, follow the instructions below: (a) Deactivate the sources and nd RN by circuit reduction techniques. (b) Find iN with sources activated. (2) If the network contains resistors, independent and dependent sources, follow the steps given below: (a) Determine the short-circuit current iN with all sources activated.
Rt

Circuit Theorems

189

(b) Find the open-circuit voltage voc . (c)


Rt

= RN =

voc iN

(3) If the network contains only resistors and dependent sources, follow the procedure described below: (a) Note that iN = 0. (b) Connect 1A current source to the terminals a (c)
Rt b

vab

and nd vab .

1
voc isc

Note: Also, since vt = voc and iN = isc


Rt

= RN

The opencircuit and shortcircuit test are sufcient to nd any Thevenin or Norton equivalent.
3.3.1 PROOF OF THEVENINS AND NORTONS THEOREMS

The principle of superposition is employed to provide the proof of Thevenins and Nortons theorems.
Derivation of Thevenins theorem:

Let us consider a linear circuit having two accessible terminals x y and excited by an external current source i. The linear circuit is made up of resistors, dependent and independent sources. For the sake of simplied analysis, let us assume that the linear circuit contains only two independent voltage sources v1 and v2 and two independent current sources i1 and i2 . The terminal voltage v may be obtained, by applying the principle of superposition. That is, v is made up of contributions due to the external source and independent sources within the linear network. Hence; where
v

= a0 i + a1 v1 + a2 v2 + a3 i1 + a4 i2 = a0 i + b0 = a1 v1 + a2 v2 + a3 i1 + a4 i2 = contribution to the terminal voltage v by independent sources within the linear network.

(3.9) (3.10)

b0

Let us now evaluate the values of constants a0 and b0 . (i) When the terminals x and y are opencircuited, this fact in equation 3.10, we nd that b0 = vt .
i

= 0 and

voc

vt .

Making use of

190

Network Theory

(ii) When all the internal sources are deactivated, become v = a0 i = Rt i

b0

= 0. This enforces equation 3.10 to = Rt


Rt

a0

Vt

Figure 3.62

Current-driven circuit

Figure 3.63 Thevenins equivalent circuit of Fig. 3.62

where Rt is the equivalent resistance of the linear network as viewed from the terminals x y . Also, a0 must be Rt in order to obey the ohms law. Substuting the values of a0 and b0 in equation 3.10, we nd that v = Rt i + v1 which expresses the voltage-current relationship at terminals Thus, the two circuits of Fig. 3.62 and 3.63 are equivalent.
Derivation of Nortons theorem:
x y

of the circuit in Fig. 3.63.

Let us now assume that the linear circuit described earlier is driven by a voltage source v as shown in Fig. 3.64. The current owing into the circuit can be obtained by superposition as
i

= c0 v + d 0

(3.11)

where c0 v is the contribution to i due to the external voltage source v and d0 contains the contributions to i due to all independent sources within the linear circuit. The constants c0 and d0 are determined as follows : (i) When terminals x y are short-circuited, v = 0 and i = isc . Hence from equation (3.11), we nd that i = d0 = isc , where isc is the short-circuit current owing out of terminal x, which is same as Norton current iN Thus,
d0

iN

Figure 3.64
Voltage-driven circuit

(ii) Let all the independent sources within the linear network be turned off, that is d0 = 0. Then, equation (3.11) becomes i = c0 v

Circuit Theorems

191

For dimensional validity, c0 must have the dimension of conductance. This enforces c0 = 1 where Rt is the equivalent resistance of the linear network as seen from the terminals x Thus, equation (3.11) becomes
i Rt y.

= =

1
Rt

v v

isc

1
Rt

Figure 3.65 Nortons equivalent of


iN

voltage driven circuit

This expresses the voltage-current relationship at the terminals x y of the circuit in Fig. (3.65), validating that the two circuits of Figs. 3.64 and 3.65 are equivalents.
EXAMPLE 3.19

Find the Norton equivalent for the circuit of Fig. 3.66.

Figure 3.66
SOLUTION

As a rst step, short the terminals a b. This results in a circuit diagram as shown in Fig. 3.67. Applying KCL at node a, we get 0 4 24 3 + isc = 0

isc

= 9A

To nd RN , deactivate all the independent sources, resulting in a circuit diagram as shown in Fig. 3.68 (a). We nd RN in the same way as Rt in the Thevenin equivalent circuit. 4 12 RN = =3 4 + 12

Figure 3.67

192

Network Theory

Figure 3.68(a)

Figure 3.68(b)

Thus, we obtain Nortion equivalent circuit as shown in Fig. 3.68(b).


EXAMPLE 3.20

Refer the circuit shown in Fig. 3.69. Find the value of ib using Norton equivalent circuit. Take R = 667 .

Figure 3.69
SOLUTION

Since we want the current owing through R, remove R from the circuit of Fig. 3.69. The resulting circuit diagram is shown in Fig. 3.70. To nd iac or iN referring Fig 3.70(a) : 0 = 0A 1000 12 isc = A = 2 mA 6000
ia

Figure 3.70

Figure 3.70(a)

Circuit Theorems

193

To nd RN : The procedure for nding RN is same that of Rt in the Thevenin equivalent circuit.
Rt

= RN =

voc isc

To nd voc , make use of the circuit diagram shown in Fig. 3.71. Do not deactivate any source. Applying KVL clockwise, we get

) )
Therefore;

12 + 6000ia + 2000ia + 1000ia = 0 4 ia = A 3000 4 voc = ia 1000 = V 3 4 voc 3 RN = = = 667 isc 2 10 3

Figure 3.71

The Norton equivalent circuit along with resistor R is as shown below:


ib

isc

2mA = 1mA 2

Figure : Norton equivalent circuit with load R

EXAMPLE

3.21

Find Io in the network of Fig. 3.72 using Nortons theorem.

Figure 3.72

194

Network Theory

SOLUTION

We are interested in Io , hence the 2 k resistor is removed from the circuit diagram of Fig. 3.72. The resulting circuit diagram is shown in Fig. 3.73(a).

Figure 3.73(a)

Figure 3.73(b)

To nd iN or isc : Refer Fig. 3.73(b). By inspection, V1 = 12 V Applying KCL at node V2 :


V2 V1

6 k

V2

2 k

V2

V1

3 k

=0

Substituting V1 = 12 V and solving, we get


V2 isc

= 6V =
V1 V2

3 k

V1

4 k

= 5 mA

To nd RN : Deactivate all the independent sources (refer Fig. 3.73(c)).

Figure 3.73(c)

Figure 3.73(d)

Circuit Theorems

195

Referring to Fig. 3.73 (d), we get


RN

= Rab = 4 k [3 k + (6 k 2 k)] = 2:12 k

jj

jj

Hence, the Norton equivalent circuit along with 2 k resistor is as shown in Fig. 3.73(e).
Io

isc R

+ RN

RN

= 2:57mA
Figure 3.73(e)

EXAMPLE

3.22

Find Vo in the circuit of Fig. 3. 74.

Figure 3.74

SOLUTION

Since we are interested in Vo , the voltage across 4 k resistor, remove this resistance from the circuit. This results in a circuit diagram as shown in Fig. 3.75.

Figure 3.75

196

Network Theory

To nd isc , short the terminals a

Circuit Theorems

197

Constraint equation :
i1 i2

= 4mA
3

(3.12)

KVL around supermesh : 4+2 KVL around mesh 3 : 8

 10

i1

+4
3

 10
i3

i2

=0

(3.13)

 10 (
3 3

i3

i2 )

+2

 10 (
3

i1 )

=0

Since i3 = isc , the above equation becomes, 8

 10 (

isc

i2 )

+2

 10 (

isc

i1 )

=0

(3.14)

Solving equations (3.12), (3.13) and (3.14) simultaneously, we get isc = 0:1333 mA. To nd RN : Deactivate all the sources in Fig. 3.75. This yields a circuit diagram as shown in Fig. 3.76.

Figure 3.76

RN

= 6 k 10 k 6 10 = = 3:75 k 6 + 10

jj

Hence, the Norton equivalent circuit is as shown in Fig 3.76 (a). To the Norton equivalent circuit, now connect the 4 k resistor that was removed earlier to get the network shown in Fig. 3.76(b).

Figure 3.76(a)

198

Network Theory

Vo

= isc (RN = isc


RN

jj

R)

RN R

+R

= 258 mV

Figure 3.76(b) Norton equivalent circuit with R = 4 k

EXAMPLE

3.23
b

Find the Norton equivalent to the left of the terminals a

for the circuit of Fig. 3.77.

Figure 3.77

SOLUTION

To nd isc :

Note that vab = 0 when the terminals a Then


i

are short-circuited.

5 = 10 mA 500 Therefore, for the righthand portion of the circuit, isc = =

10i =

100 mA.

Circuit Theorems

199

To nd RN or Rt :

Writing the KVL equations for the left-hand mesh, we get 5 + 500i + vab = 0 Also for the right-hand mesh, we get
vab

(3.15)

= =

25(10i) =
vab

250i

250 Substituting i into the mesh equation (3.15), we get

Therefore

)
RN

5 + 500
voc isc

vab

250 =
vab isc

+ vab = 0 = 5V 5 = 50 0:1
vab

= Rt

The Norton equivalent circuit is shown in Fig 3.77 (a).


Figure 3.77 (a)

EXAMPLE

3.24

Find the Norton equivalent of the network shown in Fig. 3.78.

Figure 3.78

200

Network Theory

SOLUTION

Since there are no independent sources present in the network of Fig. 3.78, iN = isc = 0. To nd RN , we inject a current of 1A between the terminals a b. This is illustrated in Fig. 3.79.

Figure 3.79

Figure 3.79(a)

Norton

equivalent circuit

KCL at node 1:

)
KCL at node 2:
v2

1=

v1

100 0:03v1
v2

v1

v2

50 0:02v2 = 1

200

+ 0:1v1 = 0 50 0:08v1 + 0:025v2 = 0

v1

Solving the above two nodal equations, we get


v1

voc = 10:64 volts 10:64 Hence; R N = Rt = = = 10:64 1 1 Norton equivalent circuit for the network shown in Fig. 3.78 is as shown in Fig. 3.79(a). voc

= 10:64 volts

EXAMPLE

3.25

Find the Thevenin and Norton equivalent circuits for the network shown in Fig. 3.80 (a).

Figure 3.80(a)

Circuit Theorems

201

SOLUTION

To nd Voc : Performing source transformation on 5A current source, we get the circuit shown in Fig. 3.80 (b). Applying KVL around Left mesh :

) )

50 + 2ia

20 + 4ia = 0 70 ia = A 6

Applying KVL around right mesh: 20 + 10ia + Voc 4ia = 0


Voc

90 V
Figure 3.80(b)

To nd isc (referring Fig 3.80 (c)) : KVL around Left mesh : 50 + 2ia 6ia 4isc = 70 KVL around right mesh : 4 (isc

20 + 4 (ia

isc )

=0

ia )

+ 20 + 10ia = 0 6ia + 4isc = 20


Figure 3.80(c)

Solving the two mesh equations simultaneously, we get isc = 11:25 A voc 90 = =8 Hence, Rt = RN = isc 11:25 Performing source transformation on Thevenin equivalent circuit, we get the norton equivalent circuit (both are shown below).

Thevenin equivalent circuit

Norton equivalent circuit

202

Network Theory

EXAMPLE

3.26

If an 8 k load is connected to the terminals of the network in Fig. 3.81, VAB = 16 V. If a 2 k load is connected to the terminals, VAB = 8V. Find VAB if a 20 k load is connected across the terminals.
SOLUTION

Figure 3.81

Applying KVL around the mesh, we get (Rt + RL ) I = Voc If If


RL RL

= 2 k;

= 10 mA
I

= 10 k;

) = 6 mA )
Voc

Voc Voc

= 20 + 0:01Rt = 60 + 0:006Rt

Solving, we get Voc = 120 V, Rt = 10 k. If


RL

= 20 k;

(RL + Rt )

(20

103

120 + 10

 10 ) = 4 mA
3

3.4

Maximum Power Transfer Theorem

In circuit analysis, we are some times interested in determining the maximum power that a circuit can supply to the load. Consider the linear circuit A as shown in Fig. 3.82. Circuit A is replaced by its Thevenin equivalent circuit as seen from a and b (Fig 3.83). We wish to nd the value of the load RL such that the maximum power is delivered to it. The power that is delivered to the load is given by

Figure 3.82 Circuit A with load RL

=i


RL

Vt Rt

2
RL

+ RL

(3.16)

Circuit Theorems

203

RL .

Assuming that Vt and Rt are xed for a given source, the maximum power is a function of In order to determine the value of RL that maximizes p, we differentiate p with respect to RL and equate the derivative to zero.
dp dRL

"

= Vt2 = Rt

(Rt + RL )2 2 (Rt + RL ) =0 (RL + Rt )2 (3.17)

which yields
d p

RL

To conrm that equation (3.17) is a maximum,


2

it should be shown that

mum power is transferred to the load when RL is equal to the Thevenin equivalent resistance Rt . The maximum power transferred to the load is obtained by substituting RL = Rt in equation 3.16. Accordingly,
Pmax

2 dRL

<

0. Hence, maxi-

Figure 3.83 Thevenin equivalent circuit


is substituted for circuit A

(2RL )2

Vt R L

Vt

4RL

The maximum power transfer theorem states that the maximum power delivered by a source represented by its Thevenin equivalent circuit is attained when the load RL is equal to the Thevenin resistance Rt .
EXAMPLE 3.27

Find the load RL that will result in maximum power delivered to the load for the circuit of Fig. 3.84. Also determine the maximum power Pmax .

Figure 3.84

SOLUTION

Disconnect the load resistor RL . This results in a circuit diagram as shown in Fig. 3.85(a). Next let us determine the Thevenin equivalent circuit as seen from a b.

204

Network Theory

180 = 1A 150 + 30 i = 150 V Voc = Vt = 150


i

To nd Rt , deactivate the 180 V source. This results in the circuit diagram of Fig. 3.85(b).
Rt

= Rab = 30 150 30 150 = = 25 30 + 150

jj

Figure 3.85(a)

The Thevenin equivalent circuit connected to the load resistor is shown in Fig. 3.86. Maximum power transfer is obtained when RL = Rt = 25 : Then the maximum power is (150)2 4RL 4 25 = 2:25 Watts The Thevenin source Vt actually provides a total power of
Pmax

Vt

Figure 3.85(b)

Pt

  150 = 150  25 + 25
= 150
i

= 450 Watts Thus, we note that one-half the power is dissipated in RL .


EXAMPLE 3.28
Figure 3.86

Refer to the circuit shown in Fig. 3.87. Find the value of RL for maximum power transfer. Also nd the maximum power transferred to RL .

Figure 3.87

Circuit Theorems

205

SOLUTION

Disconnecting RL , results in a circuit diagram as shown in Fig. 3.88(a).

Figure 3.88(a)

To nd Rt , deactivate all the independent voltage sources as in Fig. 3.88(b).

Figure 3.88(b)
Rt

Figure 3.88(c)

= Rab = 6 k 6 k 6 k = 2 k

jj

jj

To nd Vt : Refer the Fig. 3.88(d). Constraint equation :


V3 V1 V2

= 12 V =3V
V2 V1 V1 V2

By inspection, KCL at supernode :

V3

6k

6k

V3

6k

V3

12 6k

V3

=0 6k 12 3 =0 6k

Figure 3.88(d)

206

Network Theory

) ) ) )

V3

3 + V3

12 + V3

15 = 0 3V3 = 30
V3

= 10

Vt

= Vab = V3 = 10 V
Figure 3.88(e)

The Thevenin equivalent circuit connected to the load resistor RL is shown in Fig. 3.88(e).
Pmax

= i2 RL

Vt

2
RL

2RL = 12:5 mW

Alternate method : It is possible to nd Pmax , without nding the Thevenin equivalent circuit. However, we have to nd Rt . For maximum power transfer, RL = Rt = 2 k. Insert the value of RL in the original circuit given in Fig. 3.87. Then use any circuit reduction technique of your choice to nd power dissipated in RL . Refer Fig. 3.88(f). By inspection we nd that, V2 = 3 V. Constraint equation :

)
V3

V3

V1 V1

= 12 = V3 12

KCL at supernode :
V2

6k

V1

V2

6k 3 +
V3

V3

2k
V3

V1

) ) ) )

V3

3
V3

6k

V3

12 6k

3 + V3

+ =0 2k 6k 15 + 3V3 + V3 12 = 0 6V3 = 30
V3 Pmax

6k 12

=0

=5 V

Figure 3.88(f)

Hence;

V3

RL

25 = 12:5 mW 2k

Circuit Theorems

207

EXAMPLE

3.29

Find RL for maximum power transfer and the maximum power that can be transferred in the network shown in Fig. 3.89.

Figure 3.89

SOLUTION

Disconnect the load resistor RL . This results in a circuit as shown in Fig. 3.89(a).

Figure 3.89(a)

To nd Rt , let us deactivate all the independent sources, which results the circuit as shown in Fig. 3.89(b). Rt = Rab = 2 k + 3 k + 5 k = 10 k For maximum power transfer RL = Rt = 10 k. Let us next nd Voc or Vt . Refer Fig. 3.89 (c). By inspection, i1 = 2 mA & i2 = 1 mA.

208

Network Theory

Applying KVL clockwise to the loop 5 k

) 5  10 1  10 ) )
3

! 3 k ! 2 k ! , we get ) + 2k  + = 0 5k  + 3k (   1  10 2  10 +3  10 2  10 +2  10
a b i2 i1 i2 i1 Vt

Figure 3.89(b)

+ Vt = 0

4 + Vt = 0
Vt

= 18 V:

The Thevenin equivalent circuit with load resistor RL is as shown in Fig. 3.89 (d). 18 = 0:9 mA i = (10 + 10) 103 Then,

Pmax

= PL = (0:9 mA)2 = 8:1 mW

 10 k

Figure 3.89(c)

Figure 3.89(d)

EXAMPLE

3.30

Find the maximum power dissipated in RL . Refer the circuit shown in Fig. 3.90.

Figure 3.90

Circuit Theorems

209

SOLUTION

Disconnecting the load resistor RL from the original circuit results in a circuit diagram as shown in Fig. 3.91.

Figure 3.91

As a rst step in the analysis, let us nd Rt . While nding Rt , we have to deactivate all the independent sources. This results in a network as shown in Fig 3.91 (a) :

Figure 3.91(a)
Rt

= Rab = [140 60 ] + 8 140 60 = + 8 = 50 : 140 + 60

jj

For maximum power transfer, RL = Rt = 50 . Next step in the analysis is to nd Vt . Refer Fig 3.91(b), using the principle of current division,
i1

i2

 + 20  170 = = 17 A 170 + 30  = 20  30 =
=
i R2 R1 R2 i R1 R1

+ R2

170 + 30

600 = 3A 200

Figure 3.91(a)

210

Network Theory

Applying KVL clockwise to the loop comprising of 50 50i2

) 50(3) )
=

10i1 + 8

0+

! 10 ! 8 !

b,

we get

Vt

=0 = 20 V
RL

10 (17) + Vt = 0
Vt

The Thevenin equivalent circuit with load resistor as shown in Fig. 3.91(c). 20 = 0:2A 50 + 50 2 Pmax = iT 50 = 0:04 50 = 2 W
iT

is
Figure 3.91(c)

EXAMPLE

3.31
RL

Find the value of nd Pmax .

for maximum power transfer in the circuit shown in Fig. 3.92. Also

Figure 3.92
SOLUTION

Disconnecting RL from the original circuit, we get the network shown in Fig. 3.93.

Figure 3.93

Circuit Theorems

211

Let us draw the Thevenin equivalent circuit as seen from the terminals a b and then insert the value of RL = Rt between the terminals a b. To nd Rt , let us deactivate all independent sources which results in the circuit as shown in Fig. 3.94.

Figure 3.94
Rt

= Rab

=8 2 8 2 = = 1:6 8+2 Next step is to nd Voc or Vt . By performing source transformation on the circuit shown in Fig. 3.93, we obtain the circuit shown in Fig. 3.95.

jj 

Figure 3.95

Applying KVL to the loop made up of 20 V

! 3 ! 2 ! 10 V ! 5 ! 30 V, we get
10 30 = 0 60 i = = 6A 10

20 + 10i

212

Network Theory

Again applying KVL clockwise to the path 2 2i 10 Vt = 0

! 10 V !

b,

we get

Vt

= 2i

10

= 2(6)

10 = 2 V

The Thevenin equivalent circuit with load resistor RL is as shown in Fig. 3.95 (a).
Pmax

= i2 T RL =
Vt

4Rt

= 625 mW

Figure 3.95(a) Thevenin equivalent


circuit

EXAMPLE

3.32

Find the value of RL for maximum power transfer. Hence nd Pmax .

Figure 3.96
SOLUTION

Removing RL from the original circuit gives us the circuit diagram shown in Fig. 3.97.

Figure 3.97

To nd Voc : KCL at node A :

)
Hence;

ia

0:9 + 10i0 a=0


ia Voc
0

= 0:1 A = 3 10i0 a =3

 10  0 1 = 3 V
:

Circuit Theorems

213

To nd Rt , we need to compute isc with all independent sources activated. KCL at node A:

)
Hence
isc

ia

00

0:9 + 10ia 00 = 0
00

= 10ia = 10
Rt

Voc isc

01=1A
:

ia

00

= 0:1 A

3 =3 1

Hence, for maximum power transfer RL = Rt = 3 . The Thevenin equivalent circuit with RL = 3 inserted between the terminals a b gives the network shown in Fig. 3.97(a).
iT Pmax

= (0:5)2

= i2 T RL

3 = 0:5 A 3+3

3
Figure 3.97(a)

= 0.75 W
EXAMPLE 3.33

Find the value of RL in the network shown that will achieve maximum power transfer, and determine the value of the maximum power.

Figure 3.98(a)
SOLUTION

Removing RL from the circuit of Fig. 3.98(a), we get the circuit of Fig 3.98(b). Applying KVL clockwise we get 0 12 + 2 103 i + 2Vx =0

Also Hence;

Vx

=1

 10
3i 3

12 + 2
i

 10 12 = 4  10

+2 1

 10

3i

=0
Figure 3.98(b)

= 3 mA

214

Network Theory

Applying KVL to loop 1 k

!2 ! 1  10
Vx


b i

a,

we get
Vt

Vt

 10 + 2 1  10  = 1  10 + 2  10  = 3  10 3  10
=1
3
i

0 + 2Vx

=0

3
i

=9V To nd Rt , we need to nd isc . While nding isc , none of the independent sources must be deactivated. Applying KVL to mesh 1:

) ) )
1

12 + Vx 00 + 0 = 0 1

 10  10

Vx

00

= 12 = 12

i1

i1

= 12 mA

Applying KVL to mesh 2:

 10

i2

+ 2Vx 00 = 0
3
i2 i2

= =

24 24 mA

Applying KCL at node a:


isc

= i1 =
Vt isc

i2 Voc isc

= 12 + 24 = 36 mA Hence;
Rt

9 = 36 10 = 250

For maximum power transfer, RL = Rt = 250 . Thus, the Thevenin equivalent circuit with RL is as shown in Fig 3.98 (c) : 9 9 = A 250 + 250 500 2 Pmax = iT 250
iT

9 2 500 = 81 mW =



 250

Figure 3.98 (c) Thevenin equivalent circuit

Circuit Theorems

215

EXAMPLE

3.34

The variable resistor RL in the circuit of Fig. 3.99 is adjusted untill it absorbs maximum power from the circuit. (a) Find the value of RL . (b) Find the maximum power.

Figure 3.99
SOLUTION

Disconnecting the load resistor Fig. 3.99(a).

RL

from the original circuit, we get the circuit shown in

Figure 3.99(a)

KCL at node v1 :
v1

100 2

v1

13i0 a + 5 100 2
v1

v1

v2

=0

(3.18)

Constraint equations :
ia v2 v1
0

(3.19) (applying K C L at v2 ) (3.20) (3.21)

4
va
0

0 = va

= v1

v2

(potential across 4 )

216

Network Theory

From equations (3.20) and (3.21), we have


v2 v1

) ) )
v1

4
v2 v1

= v1 = 4v1 = v2

v2

4v2 (3.22)

5v1

5v2 = 0
v1

Making use of equations (3.19) and (3.22) in (3.18), we get 100 2


v2

13

(100 2

v1 )

) ) ) )
Hence; We know that,


100) + 2 5v1

5
v1

+ 13

v1

v1

4
v1 )

=0

5 (v1

(100

500 + 2v1 20v1 = 1800


v1 vt Rt

13

=0 2 100 + 13v1 = 0

= 90 Volts = v2 = v1 = 90 Volts =
voc isc

vt

isc

The short circuit current is calculated using the circuit shown below:

Here Applying KCL at node v1 :


v1

ia

00

100 2
v1

v1

100 2
v1

v1

100 2

13ia 0 v1 + =0 5 4 (100 v1 ) 13 v1 2 + =0 5 4



Circuit Theorems

217

00 Solving we get v1 = 80 volts = va Applying KCL at node a :

v1

)
Hence;

00 + isc = va 00 + va 4 80 = + 80 = 100 A 4 voc vt = Rt =

isc

v1

isc

isc

90 = 0:9 = 100 Hence for maximum power transfer,


RL

= Rt = 0:9

The Thevenin equivalent circuit with RL = 0:9 is as shown. 90 90 = 0:9 + 0:9 1:8 2 Pmax = it 0:9
it

 90 2
1:8

 0 9 = 2250 W
:

EXAMPLE

3.35

Refer to the circuit shown in Fig. 3.100 : (a) Find the value of RL for maximum power transfer. (b) Find the maximum power that can be delivered to RL .

Figure 3.100

218

Network Theory

SOLUTION

Removing the load resistor RL , we get the circuit diagram shown in Fig. 3.100(a). Let us proceed to nd Vt .

Figure 3.100(a)

Constraint equation :
ia
0

= i1 + 20 (i1 25i1

i3

KVL clockwise to mesh 1 :

) ) ) )

200 + 1 (i1

i2 )

i3 ) i2

+ 4i1 = 0 20i3 =
i1 ) i1 )

200

KVL clockwise to mesh 2 : 14i0 a + 2 (i2 14 (i1


i3 ) i3 ) i3 )

+ 1 (i2 + 1 (i2

=0 =0

+ 2 (i2

13i1 + 3i2 2 (i3


i2 )

16i3 = 0
i1 )

KVL clockwise to mesh 3 : 100 + 3i3 + 20 (i3 20i1


i1

=0

2i2 + 25i3 = 100

Solving the mesh equations, we get = 2:5A; i3 = 5A


b

Applying KVL clockwise to the path comprising of a

! 20 , we get

Vt

20i0 a=0
Vt

= 20i0 a = 20 (i1 = 150 V


i3 )

= 20 ( 2:5

5)

Circuit Theorems

219

Next step is to nd Rt .
Rt

Voc isc

Vt isc

When terminals a

00 are shorted, i00 a = 0. Hence, 14 ia is also zero.

KVL clockwise to mesh 1 :

)
KVL clockwise to mesh 2 :

200 + 1 (i1

i2 )

+ 4i1 = 0
i2

5i1

200

)
KVL clockwise to mesh 3 :

2 (i2

i3 )

+ 1 (i2 + 3i2

i1 )

=0

i1

2i3 = 0

100 + 3i3 + 2 (i3

i2 )

=0

2i2 + 5i3 = 100

220

Network Theory

Solving the mesh equations, we nd that


i3 = 60A 150 Rt = = = 2:5 isc 60 For maximum power transfer, RL = Rt = 2:5 . The Thevenin equivalent circuit with RL is as shown below : isc Vt

i1

40A; = i1

i3

= 20A;

Pmax

= i2 1 RL 150 = 2:5 + 2:5 = 2250 W

2

25
:

EXAMPLE

3.36

A practical current source provides 10 W to a 250 load and 20 W to an 80 load. A resistance RL , with voltage vL and current iL , is connected to it. Find the values of RL , vL and iL if (a) vL iL is a maximum, (b) vL is a maximum and (c) iL is a maximum.
SOLUTION

Load current calculation: 10W to 250 corresponds to iL =

10 250 =r 200 mA 20 20W to 80 corresponds to iL = 80 = 500 mA Using the formula for division of current between two parallel branches :
i2

R1

R1 RN RN

+ R2 + 250 + 80 (3.23) (3.24)

In the present context, and

0:2 = 0:5 =

IN R N IN R N

Circuit Theorems

221

Solving equations (3.23) and (3.24), we get


IN RN

= 1:7 A = 33:33

(a) If vL iL is maximum,
RL

= RN = 33:33 33:33 iL = 1:7 33:33 + 33:33 = 850 mA

vL

= iL RL = 850 = 28:33 V

 10  33 33
3
: RN RL

(b) vL = IN (RN . RL = Then, iL = 0 and

jj

RL )

is a maximum when

jj

is a maximum, which occurs when

vL

 = 1 7  33 33
= 1:7
: RN :

= 56:66 V (c)
iL

IN R N

RN

+ RL

is maxmimum when RL = 0
iL

= 1:7A and vL = 0 V

3.5

Sinusoidal steady state analysis using superposition, Thevenin and Norton equivalents

Circuits in the frequency domain with phasor currents and voltages and impedances are analogous to resistive circuits. To begin with, let us consider the principle of superposition, which may be restated as follows : For a linear circuit containing two or more independent sources, any circuit voltage or current may be calculated as the algebraic sum of all the individual currents or voltages caused by each independent source acting alone.

Figure 3.101 Thevenin equivalent circuit

Figure 3.102

Norton equivalent circuit

222

Network Theory

The superposition principle is particularly useful if a circuit has two or more sources acting at different frequencies. The circuit will have one set of impedance values at one frequency and a different set of impedance values at another frequency. Phasor responses corresponding to different frequencies cannot be superposed; only their corresponding sinusoids can be superposed. That is, when frequencies differ, the principle of superposition applies to the summing of time domain components, not phasors. Within a component, problem corresponding to a single frequency, however phasors may be superposed. Thevenin and Norton equivalents in phasor circuits are found exactly in the same manner as described earlier for resistive circuits, except for the subtitution of impedance Z in place of resistance R and subsequent use of complex arithmetic. The Thevenin and Norton equivalent circuits are shown in Fig. 3.101 and 3.102. The Thevenin and Norton forms are equivalent if the relations (a) Zt = ZN (b) Vt = ZN IN

hold between the circuits. A step by step procedure for nding the Thevenin equivalent circuit is as follows: 1. Identify a seperate circuit portion of a total circuit. 2. Find Vt = Voc at the terminals. 3. (a) If the circuit contains only impedances and independent sources, then deactivate all the independent sources and then nd Zt by using circuit reduction techniques. (b) If the circuit contains impedances, independent sources and dependent sources, then either shortcircuit the terminals and determine Isc from which Zt = Voc Isc

or deactivate the independent sources, connect a voltage or current source at the terminals, and determine both V and I at the terminals from which Zt = V I

A step by step procedure for nding Norton equivalent circuit is as follows: (i) Identify a seperate circuit portion of the original circuit. (ii) Short the terminals after seperating a portion of the original circuit and nd the current through the short circuit at the terminals, so that IN = Isc . (iii) (a) If the circuit contains only impedances and independent sources, then deactivate all the independent sources and then nd ZN = Zt by using circuit reduction techniques. (b) If the circuit contains impedances, independent sources and one or more dependent Voc sources, nd the opencircuit voltage at the terminals, Voc , so that ZN = Zt = : Isc

Circuit Theorems

223

EXAMPLE

3.37
a b

Find the Thevenin and Norton equivalent circuits at the terminals Fig. 3.103.

for the circuit in

Figure 3.103
SOLUTION

As a rst step in the analysis, let us nd Vt :

Using the principle of current division, 8 (4 /0 ) 32 = 8 + j 10 j 5 8 + j5 j 320 Vt = Io (j 10) = = 33:92 /58 V 8 + j5 Io = To nd Zt , deactivate all the independent sources. This results in a circuit diagram as shown in Fig. 3.103 (a).

Figure 3.103(a)

Figure 3.103(b) Thevenin equivalent circuit

224

Network Theory

Zt = j 10 (8 j 5) (j 10)(8 j 5) = j 10 + 8 j5 = 10 /26 The Thevenin equivalent circuit as viewed from the terminals a b is as shown in Fig 3.103(b). Performing source transformation on the Thevenin equivalent circuit, we get the Norton equivalent circuit. Vt 33:92 /58 = Zt 10 /26 = 3:392 /32 A

jj

Figure : Norton equivalent circuit

IN =

ZN = Zt = 10 /26
EXAMPLE 3.38

Find vo using Thevenins theorem. Refer to the circuit shown in Fig. 3.104.

Figure 3.104

SOLUTION

Let us convert the circuit given in Fig. 3.104 into a frequency domain equiavalent or phasor circuit (shown in Fig. 3.105(a)). ! = 1 10 cos (t 5 sin (t + 30 ) = 5 cos (t
L C

45 ) 60 )
j !L

= 1H

! = 1F !

! !
=j =

10 / 45 V 5 / 60 V

1
j !C

11= 1 1 11 = 1
j j

Circuit Theorems

225

Figure 3.105(a)

Disconnecting the capicator from the original circuit, we get the circuit shown in Fig. 3.105(b). This circuit is used for nding Vt .

Figure 3.105(b)

KCL at node a : Vt Solving; 10 / 45 Vt 5 / 60 + =0 3 j1 Vt = 4:97 / 40:54


Figure 3.105(c)

To nd Zt deactivate all the independent sources in Fig. 3.105(b). This results in a network as shown in Fig. 3.105(c) : Zt = Zab = 3 j 1 j3 3 = = (1 + j 3) 3+j 10

jj

The Thevenin equivalent circuit along with the capicator is as shown in Fig 3.105(d). Vo = Vt ( j 1) Zt j 1 4:97 / 40:54 ( = 0:3(1 + j 3) j 1 = 15:73 /247:9 V

j 1)

Hence;

vo

= 15:73 cos (t + 247:9 ) V

Figure 3.105(d) Thevenin equivalent circuit

226

Network Theory

EXAMPLE

3.39

Find the Thevenin equivalent circuit of the circuit shown in Fig. 3.106.

Figure 3.106

SOLUTION

Since terminals a

are open, Va = Is

 10
Va = 0

= 20 /0 V Applying KVL clockwise for the mesh on the right hand side of the circuit, we get 3Va + 0 (j 10) + Voc Voc = 4Va = 80 /0 V Let us transform the current source with 10 parallel resistance to a voltage source with 10 series resistance as shown in gure below :

To nd Zt , the independent voltage source is deactivated and a current source of I A is connected at the terminals as shown below :

Circuit Theorems

227

Applying KVL clockwise we get,

)
Since we get Hence;

V0 a

3V0 a 4Va
0

j 10I j 10I

+ Vo = 0 + Vo = 0 V0 a = 10I
j 10I

40I

= Vo Vo = 40 + j 10 Zt = I
Figure 3.106(a)

Hence the Thevenin equivalent circuit is as shown in Fig 3.106(a) :

EXAMPLE

3.40

Find the Thevenin and Norton equivalent circuits for the circuit shown in Fig. 3.107.

Figure 3.107
SOLUTION

The phasor equivalent circuit of Fig. 3.107 is shown in Fig. 3.108. KCL at node a : Voc 2Voc
j 10

Voc =

Voc =0 j5 100 100 j = / 90 V 3 3 10 +

228

Network Theory

Figure 3.108

To nd Isc , short the terminals a

of Fig. 3.108 as in Fig. 3.108(a).

Figure 3.108 (a)

Figure 3.108 (b)

Since Voc = 0, the above circuit takes the form shown in Fig 3.108 (b). Isc = 10 /0 A 100 / 90 Voc 10 3 Hence; Zt = = = / 90  Isc 10 /0 3 The Thevenin equivalent and the Norton equivalent circuits are as shown below.

Figure Thevenin equivalent

Figure

Norton equivalent

EXAMPLE

3.41

Find the Thevenin and Norton equivalent circuits in frequency domain for the network shown in Fig. 3.109.

Circuit Theorems

229

Figure 3.109

SOLUTION

Let us nd Vt = Vab using superpostion theorem. (i) Vab due to 100 /0

I1 = Vab1

100 /0 100 = A j 300 + j 100 j 200 = I1 (j 100) 100 (j 100) = 50 /0 Volts = j 200

(ii) Vab due to 100 /90

230

Network Theory

Hence;

100 /90 j 100 j 300 Vab2 = I2 ( j 300) 100 /90 = ( j 100 j 300 Vt = Vab1 + Vab2 I2 = = 50 + j 150

j 300)

= j 150 V

= 158:11 /108:43 V To nd Zt , deactivate all the independent sources.

Zt = j 100 j 300 j 100( j 300) = = j 150 j 100 j 300 Hence the Thevenin equivalent circuit is as shown in Fig. 3.109(a). Performing source transformation on the Thevenin equivalent circuit, we get the Norton equivalent circuit. 158:11 /108:43 Vt = = 1:054 /18:43 A Zt 150 /90 ZN = Zt = j 150 IN = The Norton equivalent circuit is as shown in Fig. 3.109(b).

jj

Figure 3.109(a)

Figure 3.109(b)

Circuit Theorems

231

3.6

Maximum power transfer theorem

We have earlier shown that for a resistive network, maximum power is transferred from a source to the load, when the load resistance is set equal to the Thevenin resistance with Thevenin equivalent source. Now we extend this result to the ac circuits.

Figure 3.110 Linear circuit

Figure 3.111

Thevenin equivalent circuit

In Fig. 3.110, the linear circuit is made up of impedances, independent and dependent sources. This linear circuit is replaced by its Thevenin equivalent circuit as shown in Fig. 3.111. The load impedance could be a model of an antenna, a TV, and so forth. In rectangular form, the Thevenin impedance Zt and the load impedance ZL are Zt = Rt + jXt and The current through the load is I= Vt Vt = Zt + ZL (Rt + jXt ) + (RL + jXL ) ZL = RL + jXL

The phasors I and Vt are the maximum values. The corresponding RM S values are obtained by dividing the maximum values by 2. Also, the RM S value of phasor current owing in the load must be taken for computing the average power delivered to the load. The average power delivered to the load is given by
P

= =

1 2 I RL 2

jj

(Rt + RL ) (Xt + XL )2

jV j
t 2

2 RL 2

(3.25)

Our idea is to adjust the load parameters RL and XL so that P is maximum. To do this, we get
@P @RL

and

@P

@XL

equal to zero.

232

Network Theory

@P @XL

j j
Vt

RL (Xt

+ XL )

@P @RL

j j
Vt

(Rt + RL )2 + (Xt + XL )2
2

i2
2RL (Rt + RL )

(Rt + RL )2 + (Xt + XL )2

2 (Rt + RL )2 + (Xt + XL )2
@P @XL XL @P @RL RL

i2

Setting

= 0 gives =
Xt

(3.26)

and Setting

= 0 gives

2 Rt

+ (Xt + XL )2

(3.27)

Combining equations (3.26) and (3.27), we can conclude that for maximum average power transfer, ZL must be selected such that XL = Xt and RL = Rt . That is the maximum average power of a circuit with an impedance Zt that is obtained when ZL is set equal to complex conjugate of Zt . Setting RL = Rt and XL = Xt in equation (3.25), we get the maximum average power as 8Rt In a situation where the load is purely real, the condition for maximum power transfer is obtained by putting XL = 0 in equation (3.27). That is,
RL P

j j =
Vt

2 Rt

+ Xt2 = Zt

j j

Hence for maximum average power transfer to a purely resistive load, the load resistance is equal to the magnitude of Thevenin impedance.
3.6.1 Maximum Power Transfer When Z is Restricted

Maximum average power can be delivered to ZL only if ZL = Z t . There are few situations in which this is not possible. These situations are described below : (i)
RL and XL may be restricted to a limited range of values. With this restriction, choose q XL as close as possible to Xt and then adjust RL as close as possible to

2 Rt

+ (XL + Xt )2 :

(ii) Magnitude of ZL can be varied but its phase angle cannot be. Under this restriction, greatest amount of power is transferred to the load when [ZL ] = Zt .

j j

 Zt is the complex conjugate of Zt .

Circuit Theorems

233

EXAMPLE

3.42

Find the load impedance that transfers the maximum power to the load and determine the maximum power quantity obtained for the circuit shown in Fig. 3.112.

Figure 3.112
SOLUTION

We select, ZL = Z t for maximum power transfer. Hence ZL = 5 + j 6 10 /0 = 1 /0 I= 5+5

Hence, the maximum average power transfered to the load is


P

1 2 I RL 2 1 = (1)2 5 = 2:5 W 2 =

jj

EXAMPLE

3.43

Find the load impedance that transfers the maximum average power to the load and determine the maximum average power transferred to the load ZL shown in Fig. 3.113.

Figure 3.113

234

Network Theory

SOLUTION

The rst step in the analysis is to nd the Thevenin equivalent circuit by disconnecting the load ZL . This leads to a circuit diagram as shown in Fig. 3.114.

Figure 3.114

Hence

Vt = Voc = 4 /0


3

= 12 /0 Volts(RMS) To nd Zt , let us deactivate all the independent sources of Fig. 3.114. This leads to a circuit diagram as shown in Fig 3.114 (a): Zt = 3 + j 4

Figure 3.114 (a)

Figure 3.115

The Thevenin equivalent circuit with ZL is as shown in Fig. 3.115. j 4. For maximum average power transfer to the load, ZL = Z t =3 It = 12 /0 3 + j4 + 3
j4

= 2 /0 A(RMS)

Hence, maximum average power delivered to the load is


P

jj
It

RL

= 4(3) = 12 W

1 It may be noted that the scaling factor is not taken since the phase current is already 2 expressed by its RM S value.

Circuit Theorems

235

EXAMPLE

3.44

Refer the circuit given in Fig. 3.116. Find the value of RL that will absorb the maximum average power.

Figure 3.116

SOLUTION

Disconnecting the load resistor RL from the original circuit diagram leads to a circuit diagram as shown in Fig. 3.117.

Figure 3.117

Vt = Voc = I1 (j 20) 150 /30 j 20 = (40 j 30 + j 20) = 72:76 /134 Volts:

To nd Zt , let us deactivate all the independent sources present in Fig. 3.117 as shown in Fig 3.117 (a). Zt = (40 j 30) j 20 j 20 (40 j 30) = = (9:412 + j 22:35) j 20 + 40 j 30

jj

236

Network Theory

The Value of RL that will absorb the maximum average power is


RL

= Zt =

j j

(9:412)2 + (22:35)2

= 24:25 The Thevenin equivalent circuit with RL inserted is as shown in Fig 3.117 (b). Maximum average power absorbed by RL is
Pmax

1 2

jj
It

RL

Figure 3.117 (a)

where

72:76 /134 (9:412 + j 22:35 + 24:25) = 1:8 /100:2 A 1 2 Pmax = (1:8) 24:25 2 = 39:29 W It =

Figure 3.117 (b) Thevenin equivalent circuit

EXAMPLE

3.45

For the circuit of Fig. 3.118: (a) what is the value of ZL that will absorb the maximum average power? (b) what is the value of maximum power?

Figure 3.118

SOLUTION

Disconnecting ZL from the original circuit we get the circuit as shown in Fig. 3.119. The rst step is to nd Vt .

Circuit Theorems

237

Vt = Voc = I1 (

j 10)

120 /0 ( 10 + j 15 j 10

j 10)

= 107:33 / 116:57 V The next step is to nd Zt . This requires deactivating the independent voltage source of Fig. 3.119. Zt = (10 + j 15) ( = =8
j 10 (10 j 10

Figure 3.119

jj

j 10)

+ j 15) + 10 + j 15

j 14

The value of ZL for maximum average power absorbed is Z t = 8 + j 14 The Thevenin equivalent circuit along with ZL = 8 + j 14 is as shown below:

Hence;

107:33 / 116:57 8 j 14 + 8 + j 14 107:33 = / 116:57 A 16 1 2 Pmax = It R L 2   1 107:33 2 8 = 2 16 = 180 Walts It =

jj

238

Network Theory

EXAMPLE

3.46

(a) For the circuit shown in Fig. 3.120, what is the value of ZL that results in maximum average power that will be transferred to ZL ? What is the maximum power ? (b) Assume that the load resistance can be varied between 0 and 4000 and the capacitive reactance of the load can be varied between 0 and 2000 . What settings of RL and XC transfer the most average power to the load ? What is the maximum average power that can be transferred under these conditions?

Figure 3.120
SOLUTION

(a) If there are no constraints on RL and XL , the load indepedance ZL = Z t = (3000 j 4000) . Since the voltage source is given in terms of its RM S value, the average maximum power delivered to the load is
Pmax

= It

where

)
(b) Since
XC RL

10 /0 3000 + j 4000 + 3000 10 = A 2 3000 2 Pmax = It RL 100 = 3000 4 (3000)2 = 8:33 mW It =

jj

RL j 4000

 jj 
q

and

XC

2000 . Next we set RL as close to


RL

are restricted, we rst qset =

XC

as close to
2

4000 as possible; hence

2 Rt

+ (XC + XL ) as possible.

Thus,

30002 + ( 2000 + 4000)2 = 3605:55


RL

Since RL can be varied between 0 to 4000 , we can set adjusted to a value ZL = 3605:55 j 2000 :

to 3605:55 . Hence ZL is

Circuit Theorems

239

It =

10 /0 3000 + j 4000 + 3605:55 = 1:4489 / 16:85 mA

j 2000

The maximum average power delivered to the load is


Pmax

= It

jj

RL

= 1:4489

= 7:57 mW

 10

3 2

 3605 55
: RL

Note that this is less than the power that can be delivered if there are no constraints on and XL .
EXAMPLE 3.47

A load impedance having a constant phase angle of 45 is connected across the load terminals a and b in the circuit shown in Fig. 3.121. The magnitude of ZL is varied until the average power delivered, which is the maximum possible under the given restriction. (a) Specify ZL in rectangular form. (b) Calculate the maximum average power delivered under this condition.

Figure 3.121

SOLUTION

Since the phase angle of ZL is xed at that

45 , for maximum power transfer to ZL it is mandatory

jZ j = p jZ j
L t

(3000)2 + (4000)2

= 5000 : Hence; ZL = ZL / 45 5000 5000 j = 2 2

j j p

240

Network Theory

It =

10 /0 (3000 + 3535:53) + j (4000

3535:53)

= 1:526 / 4:07 mA
Pmax

= It

jj

RL

= 1:526

 10

3 2

 3535 53
:

= 8:23 mW This power is the maximum average power that can be delivered by this circuit to a load impedance whose angle is constant at 45 . Again this quantity is less than the maximum power that could have been delivered if there is no restriction on ZL . In example 3.46 part (a), we have shown that the maximum power that can be delivered without any restrictions on ZL is 8.33 mW.

3.7

Reciprocity theorem

The reciprocity theorem states that in a linear bilateral single source circuit, the ratio of excitation to response is constant when the positions of excitation and response are interchanged.
Conditions to be met for the application of reciprocity theorem :

(i) The circuit must have a single source. (ii) Initial conditions are assumed to be absent in the circuit. (iii) Dependent sources are excluded even if they are linear. (iv) When the positions of source and response are interchanged, their directions should be marked same as in the original circuit.
EXAMPLE 3.48

Find the current in 2 resistor and hence verify reciprocity theorem.

Figure 3.122

Circuit Theorems

241

SOLUTION

The circuit is redrawn with markings as shown in Fig 3.123 (a).

Figure 3.123 (a)

Then;

R1 R2 R3

= (8

+2

= 1:6

= 1:6 + 4 = 5:6

= (5:6 1 + 4 1 ) 1 = 2:3333 20 = 3:16 A Current supplied by the source = 4 + 2:3333 4 = 1:32 A Current in branch ab = Iab = 3:16 4 + 4 + 1: 6 8 = 1:05 A Current in 2; I1 = 1:32 10

Verication using reciprocity theorem The circuit is redrawn by interchanging the position of excitation and response as shown in Fig 3.123 (b).

Figure 3.123 (b)

Solving the equivalent resistances,


R4

= 2;

R5

= 6;

R6

= 3:4286

Now the current supplied by the source = 20 = 3:6842A 3:4286 + 2

242

Network Theory

Therefore,
Icd

= 3:6842

8  8+ = 2 1053A 6
:

I2

2:1053 = 1:05A 2

As I1 = I2 = 1:05 A, reciprocity theorem is veried.


EXAMPLE 3.49

In the circuit shown in Fig. 3.124, nd the current through 1:375 resistor and hence verify reciprocity theorem.

Figure 3.124
SOLUTION

Figure 3.125

KVL clockwise for mesh 1 : 6:375I1 KVL clockwise for mesh 2 : 2I1 + 14I2 KVL clockwise for mesh 3 : 3I1 10I2 + 14I3 = 10 10I3 = 0 2I2 3I3 = 0

Circuit Theorems

243

Putting the above three mesh equations in matrix form, we get

2 4

6:375 2 3

2 14 10

3 10 14
I1

32 54

I1 I2 I3

3 2 5=4

0 0 10

3 5

Using Cramers rule, we get = 2A Negative sign indicates that the assumed direction of current ow should have been the other way. Verication using reciprocity theorem : The circuit is redrawn by interchanging the positions of excitation and response. The new circuit is shown in Fig. 3.126.

Figure 3.126

The mesh equations in matrix form for the circuit shown in Fig. 3.126 is

2 4

6:375 2 3

2 3 14 10 10 14
I3
0

32 54

I1 I2 I3
0 0

3 2 5=4

10 0 0

3 5

Using Cramers rule, we get = 2A


0 = Since I1 = I3

2 A, the reciprocity theorem is veried.

EXAMPLE

3.50

Find the current Ix in the j 2 impedance and hence verify reciprocity theorem.

Figure 3.127

244

Network Theory

SOLUTION

With reference to the Fig. 3.127, the current through j 2 impepance is found using series parallel reduction techniques. Total impedance of the circuit is ZT = (2 + j 3) + ( j 5) (3 + j 2) ( j 5)(3 + j 2) = 2 + j3 + j5 + 3 + j2 = 6:537 /19:36 The total current in the network is IT = 36 /0 6:537 /19:36 = 5:507 / 19:36 A

jj

Using the principle of current division, we nd that Ix = IT ( j 5) j5 + 3 + j2 = 6:49 / 64:36 A

Verication of reciprocity theorem : The circuit is redrawn by changing the positions of excitation and response. This circuit is shown in Fig. 3.128. Total impedance of the circuit shown in Fig. 3.128 is Z0 T = (3 + j 2) + (2 + j 3) ( j 5) (2 + j 3) ( j 5) = (3 + j 2) + 2 + j3 j5 = 9:804 /19:36 The total current in the circuit is I0 T = 36 /0
0 ZT

jj

= 3:672 / 19:36 A

Figure 3.128

Using the principle of current division, I0 T ( j 5) = 6:49 / 64:36 A j5 + 2 + j3 It is found that Ix = Iy , thus verifying the reciprocity theorem. Iy =
EXAMPLE 3.51

Refer the circuit shown in Fig. 3.129. Find current through the ammeter, and hence verify reciprocity theorem.

Circuit Theorems

245

Figure 3.129

SOLUTION

To nd the current through the ammeter : By inspection the loop equations for the circuit in Fig. 3.130 can be written in the matrix form as

2 4

16 1 10

1 26 20

10 20 30

32 54

I1 I2 I3

3 2 5=4

0 0 50

3 5

Using Cramers rule, we get


I1 I2

= 4:6 A = 5:4 A
I1

Hence current through the ammeter = I2 Verication of reciprocity theorem:

= 5:4 4:6 = 0:8A.

Figure 3.130

The circuit is redrawn by interchanging the positions of excitation and response as shown in Fig. 3.131. By inspection the loop equations for the circuit can be written in matrix form as

2 4

15 0 10

0 25 20

10 20 31

32 54

I1 I2 I3
0 0

3 2 5=4

50 50 0

3 5

Using Cramers rule we get


I3
0

= 0:8 A

Figure 3.131

246

Network Theory

Hence, current through the Ammeter = 0.8 A. It is found from both the cases that the response is same. Hence the reciprocity theorem is veried.
EXAMPLE 3.52

Find current through 5 ohm resistor shown in Fig. 3.132 and hence verify reciprocity theorem.

Figure 3.132
SOLUTION

By inspection, we can write

2 4

12 0 0 2 + j 10 2 2

2 2 9

32 54

I1 I2 I3

3 2 5=4

20 20 0

3 5

Using Cramers rule, we get I3 = 0:5376 / 126:25 A Hence, current through 5 ohm resistor = 0:5376 / 126:25 A Verication of reciprocity theorem: The original circuit is redrawn by interchanging the excitation and response as shown in Fig. 3.133.

Figure 3.133

Circuit Theorems

247

Putting the three equations in matrix form, we get

2 4

12 0 0 2 + j 10 2 2

2 2 9

32 56 4

I0 1 I0 2 I0 3

3 2 7 5=4

0 0 20

3 5

Using Cramers rule, we get I0 1 = 0:3876 / 2:35 A


 I0 2 = 0:456 / 78:9 A

Hence;

I0 2

I0 1=

0:3179

j 0:4335

= 0:5376 / 126:25 A The response in both cases remains the same. Thus verifying reciprocity theorem.

3.8

Millmans theorem

It is possible to combine number of voltage sources or current sources into a single equivalent voltage or current source using Millmans theorem. Hence, this theorem is quite useful in calculating the total current supplied to the load in a generating station by a number of generators connected in parallel across a busbar. En Millmans theorem states that if n number of generators having generated emfs E1 , E2 ; and internal impedances Z1 ; Z2 ; Zn are connected in parallel, then the emfs and impedances can be combined to give a single equivalent emf of E with an internal impedance of equivalent value Z.





where and

E=

where Y1 ; Y2 Yn are the admittances corresponding to the internal impedances Z1 ; Z2 and are given by 1 Z1 1 Y2 = Z2 . . . 1 Yn = Zn Y1 =



E1 Y1 + E2 Y2 + : : : + En Yn Y1 + Y2 + : : : + Yn 1 Z= Y1 + Y2 + : : : + Yn

 Z

Fig. 3.134 shows a number of generators having emfs E1 ; E2 En connected in parallel Zn are the respective internal impedances of the across the terminals x and y . Also, Z1 ; Z2 generators.





248

Network Theory

Figure 3.134

The Thevenin equivalent circuit of Fig. 3.134 using Millmans theorem is shown in Fig. 3.135. The nodal equation at x gives E1 E E2 E En E + + + =0 Z1 Z2 Zn  1 1  E E En 1 1 2 =E + + + + + + Z1 Z2 Zn Z2 Zn 1  Z 1 + En Yn = E E1 Y1 + E2 Y2 + Z



) ) )

 



where Z = Equivalent internal impedance. or

Figure 3.135
n n

[E1 Y1 + E2 Y2 +
1 1

where and
EXAMPLE 3.53

 + E Y ] = EY E Y + E Y +  + E Y E= Y Y = Y + Y +  + Y 1 1 Z= = Y Y + Y +  + Y


2 2

Refer the circuit shown in Fig. 3.136. Find the current through 10 resistor using Millmans theorem.

Figure 3.136

Circuit Theorems

249

SOLUTION

Using Millmans theorem, the circuit shown in Fig. 3.136 is replaced by its Thevenin equivalent circuit across the terminals P Q as shown in Fig. 3.137. E= E1 Y1 + E2 Y2 E3 Y3 Y1 + Y2 + Y3 22

1
5

12 1 1 1 + + 5 12 4 = 10:13 Volts 1 R= Y1 + Y2 + Y3 1 = 0:2 + 0:083 + 0:25 = 1:88 = Hence;


EXAMPLE 3.54

+ 48

1

12

1
4

Figure 3.137

IL

E R

+ 10

= 0:853 A

Find the current through (10

j 3)

using Millmans theorem. Refer Fig. 3.138.

Figure 3.138

SOLUTION

The circuit shown in Fig. 3.138 is replaced by its Thevenin equivalent circuit as seen from the terminals, A and B using Millmans theorem. Fig. 3.139 shows the Thevenin equivalent circuit along with ZL = 10 j 3 :

250

Network Theory

Figure 3.139

E=

E1 Y1 + E2 Y2 E3 Y3 Y1 + Y2 + Y3 100 /0


1
5

+ 90 /45

1
10

+ 80 /30

1
20

1 1 1 + + 5 10 20 1 = Y1 + Y2 + Y3 1
1 5

= 88:49 /15:66 V Z=R= I= Alternately, E= = +


1 10

1 20

= 2:86

88:49 /15:66 E = = 6:7 /28:79 A Z + ZL 2:86 + 10 j 3 E1 Y1 + E2 Y2 + E3 Y3 + E4 Y4 Y1 + Y2 + Y3 + Y4 100

5

+ 90 45 10 1 + 80 30 5 1 + 10 1 + 20 1 + (10 j 3)

 20
1

= 70 /12 V Therefore;
I

70 /12 10 j 3

= 6:7 /28:8 A
EXAMPLE 3.55

Refer the circuit shown in Fig. 3.140. Use Millmans theorem to nd the current through (5+j 5) impedance.

Circuit Theorems

251

Figure 3.140
SOLUTION

The original circuit is redrawn after performing source transformation of 5 A in parallel with 4 resistor into an equivalent voltage source and is shown in Fig. 3.141.

Figure 3.141

Treating the branch 5 + j 5 as a branch with Es = 0V , E1 Y1 + E2 Y2 + E3 Y3 + E4 Y4 EP Q = Y1 + Y2 + Y3 + Y4 4 2 1 + 8 3 1 + 20 4 1 = 1 2 + 3 1 + 4 1 + (5 j 5) 1 = 8:14 /4:83 V

Therefore current in (5 + j 5) is I= Alternately EP Q with (5 + j 5) open EP Q = E1 Y1 + E2 Y2 + E3 Y3 Y1 + Y2 + Y3 4 2 1 + 8 3 1 + 20 = 2 1+3 1+4 1 = 8:9231V 8:14 /4:83 = 1:15 / 40:2 A 5 + j5

4

252

Network Theory 1 1) 1

Equivalent resistance R = (2 1 + 3 Therefore current in (5 + j 5) is


I

+4

= 0:9231

8:9231 = 1:15 / 40:2 A 0:9231 + 5 + j 5

EXAMPLE

3.56

Find the current through 2 resistor using Millmans theorem. in Fig. 3.142.

Refer the circuit shown

Figure 3.142
SOLUTION

The Thevenin equivalent circuit using Millmans theorem for the given problem is as shown in Fig. 3.142(a). where E= E1 Y1 + E2 Y2 Y1 + Y2

10 /10 =

1 + 25 /90 3 + j4 1 1 + 3 + j4 5

1
5

= 10:06 /97:12 V 1 1 Z= = 1 1 Y1 + Y2 + 3 + j4 5 = 2:8 /26:56 Hence; IL = E 10:06 /97:12 = Z + 2 2:8 /26:56 + 2 = 2:15 /81:63 A

Figure 3.142(a)

Circuit Theorems

253

Reinforcement problems
R.P 3.1

Find the current in 2 resistor connected between A and B by using superposition theorem.

Figure R.P. 3.1

SOLUTION

Fig. R.P. 3.1(a), shows the circuit with 2V-source acting alone (4V-source is shorted). Resistance as viewed from 2V-source is 2 + R1 ,

where

R1

Hence; Then;

5 (1:2 + 1) 12 = 1:8592 = 14:2 2 Ia = = 0:5182 A 2 + 1:8592 12 Ib = Ia = 0:438 A 12 + 1 + 1:2 3 I1 = 0:438 = 0:2628 A 5

3  2

 +1 12

Figure R.P. 3.1(a)

With 4V-source acting alone, the circuit is as shown in Fig. R.P. 3.1(b).

Figure R.P.3.1(b)

254

Network Theory

The resistance as seen by 4V-source is 3 + R2 where


R2

14 2:7143 2 = 1:1551 = 4:7143 4 Hence; Ib = = 0:9635 A 3 + 1:1551 Ib 2:7143 = 0:555 A Thus; I2 = 4:7143 Finally, applying the principle of superposition,

 2  12

 +1 2

we get,

IAB = I1 + I2 = 0:2628 + 0:555 = 0:818 A

R.P

3.2

For the network shown in Fig. R.P. 3.2, apply superposition theorem and nd the current I.

Figure R.P. 3.2


SOLUTION

Open the 5A-current source and retain the voltage source. The resulting network is as shown in Fig. R.P. 3.2(a).

Figure R.P. 3.2(a)

Circuit Theorems

255

The impedance as seen from the voltage source is Z = (4 Hence;


j 2)

(8 + j 10) ( 8 + j8 Ia =

j 2) j 20

= 6:01 / 45

= 3:328 /135 A Z Next, short the voltage source and retain the current source. The resulting network is as shown in Fig. R.P. 3.2 (b). Here, I3 = 5A. Applying KVL for mesh 1 and mesh 2, we get 8I1 + (I1 and (I2 I1 ) ( 5) j 10 + (I1
j 2)

I2 ) (
j 2)

j 2)

=0

+ (I2

5) (

+ 4I2 = 0

Simplifying, we get (8 + j 8)I1 + j 2I2 = j 50 and Solving, we get


j 2I1

+ (4

j 4)I2

j 10

8 + j8 j2 Ib = I2 = 8 + j8
j2

j 50 j 10

j2 4 j4 = 2:897 / 23:96 A


Figure R.P. 3.2(b)

Since, Ia and Ib are owing in opposite directions, we have I = Ia Ib = 6:1121 /144:78 A


R.P 3.3

Apply superposition theorem and nd the voltage across 1 resistor. Refer the circuit shown in Fig. R.P. 3.3. Take v1 (t) = 5 cos (t + 10 ) and i2 (t) = 3 sin 2t A.

Figure R.P. 3.3

256

Network Theory

SOLUTION

To begin with let us assume v1 (t) alone is acting. Accordingly, short 10V - source and open i2 (t). The resulting phasor network is shown in Fig. R.P. 3.3(a). 5 cos (t + 10 )
L1 C1 L2 C2

! 5 /10 = 1H ! 1 = 1F ! 1 = H! 2 1 1 = F! 2


= 1rad=sec


V = j1 = =j =
j1

j !L1 j !C1

j !L2

1 2
j2

j !C2

Figure R.P. 3.3(a)

) ! 3 /0 1 = 1F ! = 1H ! 1 1 = F! 2 1 = H! 2
vb (t) !

Va = 5 /10 V
va (t) i2 (t)

= 5 cos [t + 10 ] The resulting network is shown

Let us next assume that in Fig. R.P. 3.3(b). 3 sin 2t


C1 L1 C2 L2

alone is acting.

= 2 rad=sec


A =
j

1 j !C1 2 j !L1 = j 2
j !C2

j1

Figure R.P. 3.3(b)

j !L2

= j1
j :

)
!

Vb = 3 /0

 1 +1 5 = 2 5 /33 7 15
j : : :

= 2:5 sin [2t + 33:7 ] A

Finally with 10V-source acting alone, the network is as shown in Fig. R.P. 3.3(c). Since = 0, inductors are shorted and capacitors are opened. Hence, Vc = 10 V Applying principle of superposition, we get.
v2 (t)

= va (t) = vb (t) + Vc = 5 cos (t + 10 ) + 2:5 sin (2t + 33:7 ) + 10Volts


Figure R.P. 3.3(c)

Circuit Theorems

257

R.P

3.4

Calculate the current through the galvanometer for the Kelvin double bridge shown in Fig. R.P. 3.4. Use Thevenins theorem. Take the resistance of the galvanometer as 30 .

Figure R.P. 3.4

SOLUTION

With G being open, the resulting network is as shown in Fig. R.P. 3.4(a).

Figure 3.4(a)

VA I2

Hence; Thus;

VB

10  100 = 450  100 = 20 V 9 5 =01 10 = = = 1 66 45  5 45 + 5 15+ 50 = 05+  10 = I1


: ; IB I2 : I2 : IB

: I2

= 2:5 V
VAB

= Vt = VA

VB

20 9

2:5 =

5 Volts 18

258

Network Theory

To nd Rt , short circuit the voltage source. The resulting network is as shown in Fig. R.P. 3.4(b).

Figure R.P. 3.4 (b)

Transforming the between B , E and F into an equivalent Y , we get


RB

35

50

 10 = 7

RE

35 5 = 3:5 ; 50

RF

 10 = 1
50

The reduced network after transformation is as shown in Fig. R.P. 3.4(c).

Figure R.P. 3.4(c)

Hence;

RAB

350 100 4:5 1:5 + +7 450 6 = 85:903 = Rt =

The Thevenins equivalent circuit as seen from A and B with 30 connected between A and B is as shown in Fig. R.P. 3.4(d). 5 18 IG = = 2:4mA 85:903 + 30 Negative sign implies that the current ows from B to A.
R.P 3.5

Figure R.P. 3.4(d)

Find Is and R so that the networks N1 and N2 shown in Fig. R.P. 3.5 are equivalent.

Circuit Theorems

259

Figure R.P. 3.5


SOLUTION

Transforming the current source in N1 into an equivalent voltage source, we get N3 as shown in Fig. R.P. 3.5(a). V I R = IS R (3.28) From N3 , we can write, From N2 we can write, Also from N2 ,
I V V

10Ia 2Ia 2
I

) ) )

3= 3= 3=
I

10

V V

5 (3.29)

=3

For equivalence of N1 and N2 , it is requirred that equations (3.28) and (3.29) must be same. Comparing these equations, we get
IR R

and and

IS R IS

=3 = 3 = 15A 0:2

= 0:2

Figure R.P. 3.5(a)

R.P

3.6

Obtain the Nortons equivalent of the network shown in Fig. R.P. 3.6.

Figure R.P. 3.6

260

Network Theory

SOLUTION

Terminals a and b are shorted. This results in a network as shown in Fig. R.P. 3.6(a)

Figure R.P. 3.6(a)

The mesh equations are (i) (ii) (iii) 9I1 + 0I2 6I3 = 30 (3.30) (3.31) (3.32)

0I1 + 25I2 + 15I3 = 30

)
IN

6I1 + 15I2 + 23I3 = 4VX = 4 (10I2 ) 6I1 25I2 + 23I3 = 0

Solving equations (3.30), (3.31) and (3.32), we get = Isc = I3 = 1:4706A

With terminals ab open, I3 = 0. The corresponding equations are 9I1 = 30 30 A I1 = 9 25I2 = 50 30 and I2 = A 25 30 VX = 10I2 = 10 = 12 V 25 6I1 4VX Vt = Voc = 15I2 and

Hence; Then; Hence; Thus;

=
Rt

50 V
Voc Isc

50 = 1:4706

34

Hence, Nortons equivalent circuit is as shown in Fig. R.P. 3.6(b).

Figure R.P. 3.6(b)

Circuit Theorems

261

R.P

3.7

For the network shown in Fig. R.P. 3.7, nd the Thevenins equivalent to show that
Vt

= =

V1

and

Zt

2 3

(1 + a + b
b

ab)

Figure R.P. 3.7


SOLUTION

With xy open, I1 = Hence,


Voc

V1

aV1

= Vt = aV1 + I1 + bI1 = aV1 + =


V1 V1 aV1

+b
ab]

V

aV1

[1 + a + b

With xy shorted, the resulting network is as shown in Fig. R.P. 3.7(a). Applying KVL equations, we get (i) (ii)

Figure R.P. 3.7(a)

) )

I1

+ (I1 2I1
I1 )

I2 ) I2

= V1 = V1

aV1 aV1

(3.33) (3.34)

(I2

+ I2 = aV1 + bI1

(1 + b) I1 + 2I2 = aV1

Solving equations (3.33) and (3.34), we get


Isc

= I2 =

V1

(1 + a + b 3 b

ab)

262

Network Theory
Voc Isc V1

Hence;

Zt

= =

=
b

(1 + a + b 2 V1 (1 + a + b

ab) ab)

(3

b)

3 2

R.P

3.8
I

Use Nortons theorem to determine are in ohms.

in the network shown in Fig. R.P. 3.8. Resistance Values

Figure R.P. 3.8

SOLUTION

Let IAE = x and IEF = y . Then by applying KCL at various junctions, the branch currents are marked as shown in Fig. R.P. 3.8(a). Isc = 125 x = IAB on shorting A and B . Applying KVL to the loop ABC F EA, we get 0:04x + 0:01y + 0:02 (y

20) + 0:03 (x

105) = 0 (3.35)

0:07x + 0:03y = 3:55

Applying KVL to the loop EDC EF , we get (x


y

30) 0:03 + (x

55) 0:02

(y

20) 0:02 0:05x

0:01y = 0 0:08y = 1:6 (3.36)

Circuit Theorems

263

Figure R.P. 3.8(a)

Solving equations (3.35) and (3.36), we get


x

= 46:76 A = IN = 120 = 78:24 A


x

Hence; The circuit to calculate opened.

Isc

Rt

is as shown in Fig. R.P. 3.8(b). All injected currents have been 0:03 0:05 0:08

Rt

= 0:03 + 0:04 + = 0:08875

Figure R.P. 3.8(b)

Figure R.P. 3.8(c)

264

Network Theory

The Nortons equivalent network is as shown in Fig. R.P. 3.8(c).


I

= 78:24

0 08875  0 08875 + 0 04
: : :

= 53:9A
R.P 3.9

For the circuit shown in Fig. R.P. 3.9, nd R such that the maximum power delivered to the load is 3 mW.

Figure R.P. 3.9

SOLUTION

For a resistive network, the maximum power delivered to the load is


Pmax

Vt

4Rt

The network with RL removed is as shown in Fig. R.P. 3.9(a). Let the opent circuit voltage between the terminals a and b be Vt . Then, applying KCL at node a, we get
Vt R

Vt R

Vt R

Figure R.P. 3.9(a)

=0

Simplifying we get Vt = 2 Volts With all voltage sources shorted, the resistance, found as follows: 1
Rt Rt

Rt

as viewed from the terminals, 3


R

and b is

= =

1
R R

1
R

1
R

Circuit Theorems

| 265

Hence,
R.P 3.10

Pmax =

22 3 = = 3 103 R R 4 3 R = 1 k

Refer Fig. R.P. 3.10, nd X1 and X2 interms of R1 and R2 to give maximum power dissipation in R2 .

Figure R.P. 3.10


SOLUTION

The circuit for nding Zt is as shown in Figure R.P. 3.10(a). Zt = = R1 (jX1 ) R1 + jX1

2 + jR2 X R1 X1 1 1 2 2 R1 + X1

For maximum power transfer, ZL = Z t Hence, R2 + jX2 = R2 =


2 2X R1 X1 R1 1 j 2 2 2 2 R1 + X1 R1 + X1 2 R1 X1 2 + X2 R1 1

Figure R.P. 3.10(a)

X1 = R1 X2 =

R2 R1 R2

(3.37) (3.38)

2X R1 1 2 2 R1 + X1

Substituting equation (3.37) in equation (3.38) and simplifying, we get X2 = R2 (R1 R2 )

266

Network Theory

Exercise Problems
E.P 3.1

Find ix for the circuit shown in Fig. E.P. 3.1 by using principle of superposition.

Ans : ix =
E.P 3.2

1 A 4

Figure E.P. 3.1

Find the current through branch P Q using superposition theorem.

Figure E.P. 3.2

Ans :
E.P

1.0625 A
3.3

Find the current through 15 ohm resistor using superposition theorem.

Figure E.P. 3.3

Ans :

0.3826 A

Circuit Theorems

267

E.P

3.4

Find the current through 3 + j 4 using superposition theorem.

Figure E.P. 3.4

Ans :
E.P

8.3 /85.3 A
3.5

Find the current through Ix using superposition theorem.

Figure E.P. 3.5

Ans :
E.P

3.07 / 163.12 A
3.6

Determine the current through 1 resistor using superposition theorem.

Figure E.P. 3.6

Ans :

0.406 A

268

Network Theory

E.P

3.7
b

Obtain the Thevenin equivalent circuit at terminals a

of the network shown in Fig. E.P. 3.7.

Figure E.P. 3.7

Ans :
E.P

Vt = 6.29 V, Rt = 9.43
3.8
y

Find the Thevenin equivalent circuit at terminals x

of the circuit shown in Fig. E.P. 3.8.

Figure E.P. 3.8

Ans :
E.P

Vt = 0.192 / 43.4 V, Zt = 88.7 /11.55


3.9

Find the Thevenin equivalent of the network shown in Fig. E.P. 3.9.

Figure E.P. 3.9

Ans :

Vt = 17.14 volts, Rt = 4

Circuit Theorems

269

E.P

3.10
b.

Find the Thevenin equivalent circuit across a

Refer Fig. E.P. 3.10.

Figure E.P. 3.10

Ans :
E.P

Vt =
3.11

30 V, Rt = 10 k for the network shown in Fig. E.P. 3.11.

Find the Thevenin equivalent circuit across a

Figure E.P. 3.11

Ans :
E.P

Verify your result with other methods.


3.12

Find the current through 20 ohm resistor using Norton equivalent.

Figure E.P. 3.12

Ans :

IN = 4.36 A, RN = Rt = 8.8 , IL = 1.33 A

270

Network Theory

E.P

3.13

Find the current in 10 ohm resistor using Nortons theorem.

Figure E.P. 3.13

Ans :
E.P

IN =
3.14

4 A, Rt = RN

100 = , IL = 7

0.5 A

Find the Norton equivalent circuit between the terminals a

b for the network shown in

Fig. E.P. 3.14.

Figure E.P. 3.14

Ans :
E.P

IN

= 4.98310 / 5.71 A, Zt = ZN = 3.6 /23.1

3.15
P Q

Determine the Norton equivalent circuit across the terminals Fig. E.P. 3.15.

for the network shown in

Figure E.P. 3.15

Ans :

I N = 5 A, R N = R t = 6

Circuit Theorems

271

E.P

3.16

Find the Norton equivalent of the network shown in Fig. E.P. 3.16.

Figure E.P. 3.16

Ans :
E.P

IN = 8.87 A, RN = Rt = 43.89
3.17

Determine the value of RL for maximum power transfer and also nd the maximum power transferred.

Figure E.P. 3.17

Ans :
E.P

RL = 1.92 , Pmax = 4.67 W


3.18

Calculate the value of ZL for maximum power transfer and also calculate the maximum power.

Figure E.P. 3.18

Ans :

ZL = (7.97 + j 2.16), Pmax = 0.36 W

272

Network Theory

E.P

3.19

Determine the value of RL for maximum power transfer and also calculate the value of maximum power.

Figure E.P. 3.19

Ans :
E.P

RL = 5.44 , Pmax = 2.94 W


3.20

Determine the value of ZL for maximum power transfer. What is the value of maximum power?

Figure E.P. 3.20

Ans :
E.P

ZL = 4.23 + j 1.15 , Pmax = 5.68 Watts


3.21
y.

Obtain the Norton equivalent across x

Figure E.P. 3.21

Ans :
E.P

IN = ISC = 7.35A, Rt = RN = 1.52


3.22
b

Find the Norton equivalent circuit at terminals a

of the network shown in Fig. E.P. 3.22.

Circuit Theorems

273

Figure E.P. 3.22

Ans :
E.P

IN = 1.05 /251.6 A, Zt = ZN = 10.6 /45


3.23
Y

Find the Norton equivalent across the terminals X

of the network shown in Fig. E.P. 3.23.

Figure E.P. 3.23

Ans :
E.P

IN = 7A, Zt = 8.19 / 55


3.24

Determine the current through 10 ohm resistor using Nortons theorem.

Figure E.P. 3.24

Ans :

0.15A

274

Network Theory

E.P

3.25

Determine the current I using Nortons theorem.

Figure E.P. 3.25

Ans :
E.P

Verify your result with other methods.


3.26

Find Vx in the circuit shown in Fig. E.P. 3.26 and hence verify reciprocity theorem.

Figure E.P. 3.26

Ans :
E.P

Vx = 9.28 /21.81 V
3.27

Find Vx in the circuit shown in Fig. E.P. 3.27 and hence verify reciprocity theorem.

Figure E.P. 3.27

Ans :

Vx = 10.23 Volts

Circuit Theorems

275

E.P

3.28

Find the current ix in the bridge circuit and hence verify reciprocity theorem.

Figure E.P. 3.28

Ans :
E.P

ix = 0.031 A
3.29

Find the current through 4 ohm resistor using Millmans theorem.

Figure E.P. 3.29

Ans :
E.P

I = 2.05 A
3.30

Find the current through the impedance of (10 + j 10) using Millmans theorem.

Figure E.P. 3.30

Ans :

3.384 /12.6 A

276

Network Theory

E.P

3.31

Using Millmans theorem, nd the current owing through the impedance of (4 + j 3) .

Figure E.P. 3.31

Ans :

3.64 / 15.23 A

You might also like